Explainatory Notes

Download as pdf or txt
Download as pdf or txt
You are on page 1of 33

Explanatory Notes

Exercise – 1 15. Statement B is incorrect. In this statement ‘discharged’ is


used in the context of doing everything that is necessary to
Explanatory notes for questions 1 to 25: perform and complete a particular duty. The correct usage
is to discharge a debt which means ‘pay-off one’s debts’
1. Statement D is incorrect. When we use the word ‘down’ in Therefore, we do not need the “off’ after ‘discharged’.
the context of an illness, the correct usage is ‘down with’, Choice (B)
which means ‘to have or catch an illness’ Choice (D)
16. Statement C is incorrect. When a business ‘expands’,
2. Statement C is incorrect .The correct usage is ‘live with the it means new branches have been opened, it is making
problem’, which means to get used to a condition or a more money etc. The correct usage would be “they have
situation which cannot be changed. Choice (C) expanded the business…”. The word ‘over’ is not required.
Choice (C)
3. Statement D is incorrect. When we use the word ‘matter’ in
a context of improving a situation, the correct phrasing is to 17. Statement C is incorrect. In medical parlance, a drip is a
improve ‘matters’. Choice (D) piece of equipment that passes liquid food very slowly
through a tube into a patient’s vein. The correct usage is
4. Statement C is incorrect. When we use the word ‘view’ in a ‘put on a drip’. Choice (C)
context when an intention or hope of doing something is
implied, the correct usage is ‘with a view to’. Choice (C) 18. Statement B is incorrect. The correct usage would be “gearing
up for” which means ‘to prepare yourself to do something’.
5 Statement B is incorrect .When the word ‘vote’ is used in Choice (B)
the context of being chosen for a position or an award, the
correct usage is ‘was voted’ the ‘as’ is not required. 19. Statement D is incorrect. The correct usage is ‘figured that’
Choice (B) which means ‘to think or decide that something will happen.
Choice (D)
6. Statement B is incorrect. When we use the word ‘trim’ to
indicate the act of cutting away unnecessary parts from 20. Statement B is incorrect. We use the phrase ‘filled to
something the correct usage is “trim off/away”, so the capacity’, not ‘filled in to capacity’ to indicate that a place is
correct phrasing would be “trim two centimetres off the full. Choice (B)
hem”. Choice (B)
21. Statement B is incorrect. The correct is ‘a change of focus”
7. Statement D is incorrect. The correct usage, here, would be which means to look at things in a different way.
“His past caught up with him”, which means one is forced to Choice (B)
deal with something unpleasant that one has done in the
past. Choice (D) 22. Statement C is incorrect. The correct usage would be
“an inspiration to all of us” which means’ somebody or
8. The incorrect statement is the first one. ‘Caution in the something that makes you want to be better more
sense of an official warning is used as “a caution”, so, the successful etc, Choice (C)
correct phrasing would be “got off with a caution” which
means the person was given a warning by the police for 23. Statement D is incorrect. The correct usage would be, she
committing a crime that is not too serious. managed to keep herself from laughing at him’. To ‘keep
Choice (A) yourself from something’ means to prevent yourself from
doing something. Choice (D)
9. Statement B is incorrect. The correct usage would be to
‘keep an eye out for’, because in the present context we 24. Statement D is incorrect The correct statement would be
are using the word ‘eye’ in the sense of ‘to look for ‘you can become a good player if you set your mind to it’
something [somebody] while you are doing other things’. The phrase ‘set your mind to/on something’ means to
Choice (B) decide you want to achieve something and give this all your
attention. Choice (D)
10. Statement D is incorrect. In the context of preparing food,
the correct usage of ‘fix’ is ‘fix’ something for something ‘or’ 25. Statement D is incorrect. The correct sentence would be
‘fix somebody something’. So, the correct statement would “The library has only a narrow range of books” Here, ‘narrow’
be ‘can I fix supper for you?” Choice (D) means ‘limited in variety or numbers’. Choice (D)

11. Statement B is incorrect. Since in the given sentence, Exercise – 2


‘hard” is used to indicate a situation where somebody
deliberately made a situation difficult and unpleasant for Explanatory notes for questions 1 to 25:
somebody. The correct phrasing would be “gave her a hard
1. Options (B), (C) and (D) do not convey the contrast brought
time”. Choice (B)
in by the word ‘despite’. Unprecedented means unparalleled
and wars are always destructive. Choice (A)
12. Statement C is incorrect. The correct usage would be “My
heart goes out to all those children…”. The phrase “your 2. Scepticism about ‘regular’ dialogue is improper. ‘Fear’ and
heart goes out to somebody” is used to say that you feel a ‘intending’ does not provide sense to the sentence
lot of sympathy for somebody. Choice (C) ‘Wariness’ means caution. ‘Sustained’ means maintained
for a long time. Choice (D)
13. Statement D is incorrect. When we talk about a tree losing
its leaves naturally, we use ‘shed its leaves’, or other 3. ‘Exude’ means display. It is used with emotions. Only
similar phrases. So, in the given statement we don’t need options (B) and (D) offer words that denote emotion for the
the ‘off’. Choice (D) first blank. Gentility does not arouse passion. Therefore,
option B is the correct answer. Choice (B)
14. Statement B is incorrect. In this sentence, we are talking
about the British stage as one single entity, to the whole of 4. ‘Stress’ cannot be associated with ‘excretive’ or ‘beneficial’.
the British theatre. Therefore, we need the definite ‘the’ ‘Immune system’ cannot become ‘injurious’ but only
before English stage. Choice (B) ‘impaired’. Choice (C)

Triumphant Institute of Management Education Pvt. Ltd. (T.I.M.E.) HO: 95B, 2nd Floor, Siddamsetty Complex, Secunderabad – 500 003.
Tel : 040–40088400 Fax : 040–27847334 email : info@time4education.com website : www.time4education.com SM1002114/69
5. Option (B) will be the answer. ‘Satisfactory’ does not go 24. If ethics and relationship between business and
with the idea of playing field. Moreover, it discusses environment are discussed, it can only, be ‘transcend’
‘neither’ and ‘both’. Therefore, ‘complementary’ would be which means to go beyond. ‘Domination’ goes with the idea
the appropriate word. It means something that completes. of ‘get –ahead’ mentality also. Choice (B)
Choice (B)
25. You either ‘transfuse’ or ‘infuse’ blood. Since the result is
6. The sentence provides a reason for America’s excuse. The negative, only ‘disastrous’ will fit the second blank.
correct choice is (B) ‘Demolish’ means to destroy. To destroy Choice (D)
the myths, it is ‘critical’ (or very important) to prove that
America remains open to world economy. Choice (B) Exercise – 3
7. “Concentration on the opinions of others gives the
Explanatory notes for questions 1 to 40:
‘approbation’ which means approval. Choice (C)
8. ‘Adopted’, ‘suited’ does not take the preposition ‘to’ ‘Abrupt’ 1. ‘Punctilious’ means ‘meticulous’ ‘Diligent’ ‘precise’ and
negligence is not a proper collocation. ‘Abrupt’ means thorough are synonyms in the given context. ‘Slapdash’ on
sudden and unexpected. ‘Conformed’ means suitable. the other hand means careless. Choice (D)
Choice (D)
2. Opprobrious means abusive. ‘Vituperative’, ‘denigrating’
9. Option (C) is the correct one. ‘Feasible’ means practicable and ‘pejorative’ are its synonyms. Sanguine means
‘Took off’ means to become popular. Choice (C) optimistic hence incorrect. Choice (C)

10. We are discussing ‘contradictory skills’ like ‘brood sweeps’. 3. ‘Intemperate’ means immoderate. ‘Uncurbed’ ‘unrestrained’
The contradiction to that will be attention to details. Usually and ‘unbridled’ also mean the same. ‘Perspicuous’ on the
broad sweeps cannot give attention to details and other hand means intelligible. Choice (A)
accessibility without any intrusion. Choice (A)
4. Fervent means heartfelt. ‘Ardent’, ‘passionate’ or emotional
11. Artists are not ‘averse’ to the implications of culture. We mean the same in the given context. ‘Convivial’, on the
cannot ‘discover’ the spirit of people. Option (C) is absurd. other hand, means friendly and lively. Choice (B)
Option (D) is the perfect fit. Choice (D)
5. ‘Garbled’ means mixed up. ‘Distorted’, ‘muddled’ and
12. All the words for the first blank can be used. But only ‘misconstrued’ broadly mean the same and fit in the given
‘exposed’ goes with ‘unsupported’. Choice (C) context. ‘Delayed’ on the other hand is incorrect.
Choice (D)
13. ‘Demeaning’ goes with the context of the first blank. And
‘impaired’ holds the clue for the “profound problems of the
6. ‘Baleful’ means ‘threatening’. ‘Malevolent’, ‘Antagonistic’
human mind’. Choice (B)
and ‘malicious’ are synonyms. ‘Sad’ is incorrect in the said
14. If you are disenchanted you cannot respect, if you are context. Choice (C)
happy you don’t dislike, if you are disinterested. You do not
adore. Therefore choice (C) is the correct answer. 7. ‘Abnegation’ means rejection. ‘Renunciation’, ‘repudiation’
Choice (C) and abjuration are synonyms. ‘Abuse’ is incorrect here.
Choice (C)
15. The second blank should take a negative word. Options (A)
and (B) are thus ruled out. Bust, which means bankruptcy, 8. Truculent means to be defiant. ‘Belligerent’ ‘antagonistic
does not fit the larger context. Choice (D) and ‘pugnacious’ are all synonyms. ‘Obstinate’ means
‘stubborn’ and is the wrong word here. Choice (A)
16. ‘Buoyant’ and ‘Monopolistic’ are inappropriate for the
context of the first blank. An economy is not collocated with 9. ‘Factitious’ means bogus or fake. ‘Specious’, ‘feigned’ and
‘sagging. ‘Fiercely’ indicates that the word should be ‘contrived’ are synonyms. ‘Tumultuous’, on the other hand,
‘competitive’. Choice (A) means uproarious. Choice (B)

17. ‘Refuse’ and ‘hamper’ don’t fit the blank. ‘Maiming’ means 10. ‘Wizened’ means ‘creased’ ‘withered’ and ‘lined’.
to cut off therefore ‘permanently’ does not go with it. ‘Enlightened’, on the other hand. means informed or
‘Anesthetize is to render powerless. Choice (B) learned. Choice (D)

18. The phrase “concept of liberty” points towards ‘freedom’ for 11. Subliminal means subconscious ‘Hidden’ and ‘concealed’
the first blank. Transformation also fits the second blank are synonyms. ‘Consummate’ means ‘informed’ or learned.
. Choice (C) Choice (C)

19. ‘Those arts and disciplines’ refers to nothing but humanities 12. Indispensable means ‘essential’. ‘Vital’, ‘mandatory’ and
which pertains to literature. ‘Value’ also suits the second ‘necessary’ convey the same meaning. ‘Pre–eminent’
blank. Choice (B) means ‘greatest’ or foremost. Choice (C)

20. If the performance is ‘direct’ then it will ‘increase’ in the 13. Foibles means weaknesses. ‘Flaws’, ‘shortcomings’ and
response. Same with options (B) and (C) ‘Tenuous’ means infirmities are synonyms. ‘Mistake’, on the other hand,
slight and the fourth option provides the contrast. means errors. Foibles are inherent whereas mistakes
Choice (D) happen. Choice (B)

21. Only ‘difficult’ and ‘cumbersome’ can be used in the first 14. Connote means to imply. ‘Signify’ and ‘suggest’ mean the
blank. But ‘effects’ cannot be ‘excessive’ but ‘perplexing’ same. ‘Support’, on the other hand, does not fit the context.
because of the external effects. Choice (B) Choice (C)

22. The missile is ‘hidden’ so it can only be ‘detected’. 15. ‘Circumspect’ means ‘Cautious’. ‘Wary’, ‘Discreet’ and
Exposure fits in the second blank too. Choice (D) ‘chary’ broadly mean the same. Strong is the inappropriate
word. Choice (A)
23. Only ‘widespread’ and ‘obvious’ can be used in the first
blank. Since ‘so’ is used a negative idea should be 16. ‘Protract’ means long-drawn. ‘Prolonged’, interminable and
discussed in the last part of the sentence. Therefore, ‘extended’ are synonyms ‘strenuous’, on the other hand,
‘scarcely’ which means not at all. Choice (C) means ‘arduous’ or difficult. Choice (C)
Triumphant Institute of Management Education Pvt. Ltd. (T.I.M.E.) HO: 95B, 2nd Floor, Siddamsetty Complex, Secunderabad – 500 003.
Tel : 040–40088400 Fax : 040–27847334 email : info@time4education.com website : www.time4education.com SM1002114/70
17. ‘Canker’ means an evil. ‘Scourge’,’ pestilence’ and ‘blight’ 37. Moribund means declining. Stagnating, warming or
mean the same. ‘Mystery’, on the other hand. is atrophying are synonyms. Teetering means to hesitate or
inappropriate. Choice (B) be indecisive. Choice (A)

18. ‘Turpitude’ is synonymous with ‘wickedness’ ‘depravity’ and 38. Laconic means brief. ‘Terse’, ‘succinct’ and ‘concise’ are
Vileness. ‘Deadlock’ is not synonymous with turpitude. synonyms. ‘Dull’ doesn’t fit the context. Choice (B)
Choice (D)
39. ‘Draconian’ means ‘harsh’. ‘Severe’ and ‘stringent’ mean
the same. ‘Obsolescent’, on the other hand, means dying
19. ‘Vindictive’ means revengeful. ‘Vengeful’, ‘rancorous’ and
out. Choice (D)
malevolent are synonymous. Rambling is not.
Choice (B) 40. Boomerang means rebound. ‘Ricochet’ ‘recoil’ and
‘backfire’ mean the same but fail does not fit the context.
20. ‘Raddled’ means haggard. ‘Pinched’ ‘Gaunt’ and Choice (C)
‘exhausted’ are synonymous. ‘Raffish’ on the other hand
means rakish. Choice (A) Exercise – 4

21. Portents means ‘omens’ or ‘signs. ‘Presage’ ‘prophecy’ or Explanatory notes for questions 1 to 20:
‘prognostication’ is broadly synonymous. ‘Implications’, on
1. In ‘He skipped the country….’, skip means to leave
the other hand, is inappropriate. Choice (A)
suddenly and secretly. e matches d. We find this in (C) and
(D). ‘She skips a hundred times every morning……’ refers
22. ‘Libel’ means defamation ‘Aspersions’ and ‘vilification’ are
to the game/exercise of jumping over rope. f matches c.
synonymous. ‘Oblation’ means a religious offering.
If you skip breakfast means not having breakfast. g matches
Choice (C)
b. ‘..... Skipped along the riverbank’ means walk or run play
fully – h, a. The correct matching is seen in option D.
23. Taciturn mean not talkative, ‘reserved’, ‘introverted’ and
Choice (D)
‘aloof’ are synonyms. ‘Vileness’ means nasty or
unpleasant. Choice (B) 2. In ‘….. sighted land’, sight means to see something.
e matches c. This is found in (A) and (D). When we refer to
24. ‘Abeyance’ means ‘suspension’. ‘Reserved’ and ‘deferred’ ‘….. grandmother’s sight’ we are referring to the ability to
are broadly synonymous. ‘Detestation’, on the other hand, see. f matches a. The right answer is (D). To ‘….. earn a
means intense dislike. Choice (D) sight more’ means a lot more. g matches d, ‘at the sight of
blood’ means seeing something. h matches b. Therefore,
25. Fillip means stimulus. ‘Spur’ and ‘impetus’ mean the same. option D is correct. Choice (D)
‘Cushioning’, on the other hand, mean to protect against
shock. Choice (B) 3. In ‘Her happiness showed in her behaviour’, show means
displayed. e matches b. we find this in (B), (C) and (D). In
26. Jejune means childish or ‘naive’. ‘Callow’, ‘immature’ and ‘…. put up a good show’ show refers to performance.
infantile. ‘Tremulous’ means trembling or quivering. f matches d. So the right answer is (C). The poll clearly
Choice (C) shows ….. means facts or information’s that prove
something. g matches a. TV show means a programme on
27. Obdurate means stubborn. Unmalleable, intractable and TV. h matches c. Option C is correct. Choice (C)
intransigent are synonymous. Prejudiced, which means to
be partial, is not a synonym. Choice (D) 4. In ‘…. if the outer seal is broken’, seal refers to something
with which the bottle or tin is closed to keep the content
28. Quail means to shy away. ‘Cowered’, ‘cringed’ and ‘faltered’ from coming out and anything from getting in. e matches d.
means the same. Protested means to object. Choice (A) This we find in (A) and (D). In ‘the entrance ….. was sealed
…..’, seal refers to closing the way. f matches c. The right
29. Ravenous means to be extremely hungry. ‘Famished’ and answer is (D). ‘….. a programme on seals……’ refers to the
‘starving’ are synonyms ‘Unruly’ means disorderly. sea animal. g matches a. ‘….. stamped with the Principal’s
Choice (A) seal’ means special designs that shows the authority of the
person. h matches b. The correct matching is seen in
30. Investiture means inauguration. ‘Installation’, ‘inception’ and option D. Choice (D)
‘ordination’ are synonyms. Coronation is a ceremony to
5. In the hook on the braces of her skirt…..’, braces means
crown a king. Choice (D)
the material that goes over the shoulder to hold the skirt in
place. e matches c. We find it in (A) and (D). ‘…. a neck
31. ‘Invidious’ means unfair. Iniquitous and weighted are
brace’ means something used to strengthen, stiffen or
synonyms. ‘Unassailable’ means unable to be attacked or
support. f matches d. The right answer is (D). In I braced
questioned. Choice (B)
myself ……’, brace means to prepare yourself. g matches
32. Contentious means debatable, ‘contested’ or problematic. a. Braces in connection with a dentist means wires over the
‘Impertinent’ means ‘rude’ or insolent. Choice (C) teeth. h matches b. Choice (D)

33. Unimpeachable means trustworthy. ‘Impeccable’ and 6. In ‘….. left a void in his life’, void refers to the feeling of
‘irreproachable’ mean the same. Ingenuous, on the other emptiness (here sadness). e matches d. This is found in
hand, means innocent. Choice (C) (A) and (D). ‘….. null and void’ means not legal. f matches
a. So (A) is the right answer. In ‘looked …. into the void’,
34. Cloistered means ‘sheltered’. ‘Sequestered’, reclusive and void means an empty space. g matches c. ‘void of
‘secluded’ are broadly synonyms. Gregarious, on the other expression’ means lacking in expression. h matches b.
hand, means a fun-loving or happy disposition. Choice (A)
Choice (B)
7. To bow out of politics means to come out of politics or stop
35. Imperative means something vitally important. ‘Mandatory’ being involved in politics. e matches d. This we find in
‘exigent’ and ‘indispensable’ are synonyms. ‘Desperate’ (B) and (C). In ‘….. adjust the arrow in the bow….’, bow
means reckless or outrageous. Choice (A) refers to the weapon f matches c. Hence (C) is the right
answer. In ‘…. a big bow in her hair….’ bow means a knot
36. ‘Eschew’ means to abstain from. ‘Renounce’, relinquish of cloth used decoratively. g matches b. in ‘…. bowed to the
and disavow are synonyms. ‘Augment’, on the other hand, queen …..’, bow means to bend your body as a mark of
means to increase. Choice (D) respect. Choice (C)
Triumphant Institute of Management Education Pvt. Ltd. (T.I.M.E.) HO: 95B, 2nd Floor, Siddamsetty Complex, Secunderabad – 500 003.
Tel : 040–40088400 Fax : 040–27847334 email : info@time4education.com website : www.time4education.com SM1002114/71
8. In ‘…. on the dot of eight’ dot means the exact time. 18. When you say you are poor at something it means you are
e matches b. This we find in (A), (C) and (D). Dotting the ‘i’s not good at doing it. e matches d. This is found in (B), (C)
and ‘t’s refers to paying attention to details. f matches c. and (D). When you call someone a poor kid, you are pitying
This is found in (A) and (D). ‘….. a dot on her forehead…..’ them. f matches c. Hence (C) is the right answer. When
refers to a mark or spot. g matches a. Our answer is (D), you say somebody is ‘so poor’ you mean they have little
‘….. dotted with ships ‘means full of. h matches d. money. g matches a. Finally ‘poor quality means something
Choice (D) of inferior quality. h matches b. Choice (C)

9. In ‘….never did this by design’, design means intention. e 19. If the prisoners are lashed, it means they were hit with a
matches c. This is found in (C) and (D). ‘….. designed our whip, e matches c. This is found in (A) and (D). ‘With a lash
house’ means planned or made a drawing’. f matches d. So of its tail’ means a sudden, violent movement. f matches d.
(D) is the right answer. ‘…. floral design refers to decorative So, the answer is (D). The boats were lashed to the wharf,
pattern. g matches b. ‘….. this building’s basic design’ means were tied. g watches a. ‘.… lash off the tongue .…’
means how something is planned or made. h matches a. means harsh criticism. h matches b.
Choice (D) Choice (D)

10. ‘….. to study in depth’ means to study closely looking at all 20. ‘…. point a finger at someone,’ means to blame some one.
details. e matches d. This is found in (A), (C) and (D). In e matches c. this is found in (A) and (D). ‘The whole point
planting seed at a depth of five inches, depth means the …’ is the purpose or aim. f matches d. Hence (A) is the
distance between the ground surface and the seeds.
right answer. The power point refers to an electric socket. g
f matches a. The right answer is (A). ‘… hidden depths….’
means a part of the character that is not known. g matches matches a. ‘The child pointed towards them…’ means to
c. ‘…. the depth of public feeling …..’ means the strong show someone by holding up a finger. h matches b.
emotions raised in people. h matches b. Choice (A) Choice (A)

11. In ‘Snow had drifted….’, drift means to be blown by the Exercise – 5


wind. e matches b. This is found only in (B). ‘…. drifting
around the country means wandering along aimlessly Explanatory notes for questions 1 to 20:
f matches c. ‘The drift of public opinion….’ means the flow
or change of general opinion. g matches with d. Finally, 1. In the first sentence since we are talking about living things,
balloons drifting up into the sky refers to slow movement in it should be accompanied by (the preposition by and not
air. h matches a. Choice (B) with).
In the second sentence ability is a better choice as ability is
12. When we call someone ‘a dope’, we imply that the person the thing that can be acquired and not capacity.
is stupid. e matches d. This is found only in (B). ‘…. All the
dope….’ means all the news or information. f matches c In the third sentence adoptive is appropriate as it refers to
‘the tigers ….. doped…..’ means to be given drugs to make people who have adopted someone. In this case Mr. and
them sleep. In a, ‘….dope dealer’ refers to a person who Mrs. Sharma are adoptive parents and Raju is the adopted
deals with drugs like marijuana. h matches a. child.
Choice (B) In the 4th sentence, effect is usually used as a noun and
affect is generally used as a verb. Hence in this case, affect
13. ‘…. clearly in distress’ means in pain. e matches b. This is is a better fit.
found in (B) and (D). ‘…. a distress signal’ refers to
message sent when in danger. f matches d. So, the answer In the 5th sentence, aggravated refers to a condition which
is (B) or (D). ‘…. caused his family distress’ refers to pain or is worsened further. Hence irritated is more suitable.
anxiety. g matches a. The answer is (D). ‘..… financial The correct order is abbba. Choice (C)
distress….’ refers to bad straits. h matches c.
Choice (D) 2. In the first sentence, susceptible means someone who is
prone to something. Suspected is a better fit.
14. ‘….dry and uninspiring’ means boring. Hence, e matches c.
This is found in (A), (B) and (D). ‘…. dry states….’ refers to In sentence two, the reference is to hints of connection and
states where alcohol is prohibited. f matches d. The answer eluded does not fit in. Alluded is indirect hint and hence
is (A) or (B). ‘I feel dry….’ means thirsty. g matches b. The a better fit.
answer is (A). ‘….to dry’ means to be without water. h In sentence three, the emphasis is on being on the post of
matches a. Choice (A) chairman and secretary, but not at the same time. So
alternately is a better option.
15. When we say somebody’s work lives we mean that it
continues to influence people. e matches c. This is found in In sentence 4, the reference is to a warfare technique,
(C) and (D). To live with someone is to share their home. hence guerilla is a better fit. Gorilla refers to an animal.
f matches d. So (C) is the right answer. ‘…. live animals Grille is used as a noun and grill is to be used as a verb.
means animals that are alive not dead. g matches a. Hence grill is a better option.
‘….. match shown like….’ means to be broadcast as an
event is taking place. h matches b. Choice (C) The correct order is bbabb. Choice (B)

16. When we say a road is long, we are referring to its length. e 3. In the first sentence, it means stepped up so forward is
matches c. This is found only in (A). ‘…. waiting for long’ more appropriate than foreword, which means a note
means waiting for a considerable time. f matches a. ‘…. a written at the beginning of a book.
long …. skirt’ implies a skirt reaching to the ankles.
Fortuitous means happening by chance, whereas fortunate
g matches d. ‘She longs ….’ means want something very
means lucky. Hence fortunate fits better.
badly. h matches b. Choice (A)
Forceful is in loud voice or intensely. Forcible means using
17. To bring down a peg or two means to put down a person’s excessive force. Forcible is a better option.
vanity. e matches d. This is found in (A) and (B). In hanging
To forbear is to shun or stop. Forebear refers to ancestors.
the towels on a peg, peg means fixed on wall or door to
Forbear is a better option.
hang things. f matches a. The answer is (A). In ‘…. one peg
to drink….’ peg is a measure of alcohol. g matches b. ‘peg Few is more appropriate for number than less.
them out is to fasten with pegs. h matches c. Hence, ‘a few would be a better fit. The correct order is
Choice (A) babba. Choice (D)
Triumphant Institute of Management Education Pvt. Ltd. (T.I.M.E.) HO: 95B, 2nd Floor, Siddamsetty Complex, Secunderabad – 500 003.
Tel : 040–40088400 Fax : 040–27847334 email : info@time4education.com website : www.time4education.com SM1002114/72
4. Every day is used as an adjective. Every day is used as Timbre refers to the quality of sound. Timber refers to
a noun. Hence in sentence 1, it should be every day. wood. Timbers is more apt.
Estimation should be used with rough calculation. For The order is ababa. Choice (D)
something other than that, opinion is more appropriate.
9. Tragedy refers to the downfall of the mighty. Tragic is
Era is a duration of time. Epoch is a point in time which
something that is heart wrenching. Tragic is more
brings about an era. Hence era is more appropriate.
appropriate.
Envelope is a noun and refers to the paper in which letters
Turgid means excessively complex or bombastic. Turbid
are put for posting. Envelop is a verb which means to
refers to liquid and means muddy or murky. Turgid is more
surround. Hence it should be envelope.
suitable.
Disregardless is not an appropriate choice here, regardless
Underwater is used as an adjective, and hence is more
is apt.
appropriate than under water.
The right order is baaaa. Choice (A)
Vacuous means stupid. Vacant means empty. Only the
latter fits here.
5. Observance means to act in accordance with duty or
custom. Observation is the equivalent of a viewing or A safe is a container, a vault is a room. Vault is a more
perceiving something. Observation is a better fit. suitable word here.
Obviate means to lessen, obvious means something which The order is bbabb. Choice (D)
is apparent. Obviate is the answer.
10. A valetudinarian refers to a person who is anxious about
Optimistic refers to an attitude, whereas hopeful refers to
his health, as he suffers from various medical conditions.
a particular incident or instant. In this case hopeful is a
Valedictorian is the class topper who is accorded the honor
better word.
of giving the parting speech.
Partook means to take a part and participate means to get
A verdict is given by jury. A judge either gives a judgement
involved. Participated is a better fit.
or a ruling.
Perspective refers to a view point. Prospective refers to
‘Via’ is used to talk about the route and not the mode of
possible future. Prospective is a better fit.
transport. In is a more appropriate word.
The correct sequence is bbbab. Choice (D)
Waver refers to the sideways movement either physically or
mentally. Someone is wavering if he/she is unable to take a
6. Preceding means immediately coming before. Previous
stand. waiver means to relinquish. Waver is the appropriate
means all existing prior to this. Previous is a better option.
word to be used.
Punctilious means being more focused on matters of
Xerox is a brand name. Photocopy is a more suitable word.
etiquette and protocol, Punctual means someone who
respects time and is always on time. Punctual is more apt. The order is bbbbb. Choice (B)
Pupil can be used for students of a lower grade. Students is
11. Culminates refers to reach a pinnacle. Sorting mail is not
a more suitable word.
a precursor to reaching a pinnacle. Hence, end is a more
Quote should be used as a verb, and quotation to be used apt choice.
as a noun. Quotation is more appropriate.
Connotes is indirect reference. Denotes is a direct
Rare should be used when something is not found reference. Denotes is a better choice of word.
abundantly. Scarce is to be used when something becomes
Contagious refers to disease which can be transferred
rare as a result of something and the situation is only for
through contact. Infectious refers to diseases transferred by
a short time.
a germ or parasite. Infectious is a better option.
The order is aabab. Choice (C)
On the horns of a dilemma is an idiom and hence a better fit,
7. An article is readable if the reader is interested in reading it. Discreet is to be prudent. Discrete is separate and
It is a measure of the author’s ability to generate interest. countable. Discrete is apt.
Legible refers to whether the handwriting can be
The order is bbbab. Choice (A)
comprehended or not. Readable is a better choice of word.
Raise is used when an action is done on the object. In this 12. Eponymous is the name of the character referred in the
case rise is a better option. title. Anonymous is unknown. Eponymous is more
appropriate.
Resistor is a device which helps in the passage of
electricity. Resister is someone who opposes an action. Curl means curve. Coiled is to make circles. Coiled is more
Resister is more apt. suitable.
Revue means a theatrical presentation and is always used Recalled is to call someone back. Recoiled is to carry out
as a noun. Review is more apt and it means checking. Newton’s second law of motion. Recoiled means to move
backward. Recoiled is a better fit.
Riffled means to browse through the pages of a book.
Rifled means to ransack or plunder. Rifled is apt here. Adjure is to entreat. Abjure is to renounce. abjure is more
apt.
The order is babaa. Choice (C)
Besides means in addition to, whereas beside means
alongside. Besides is more appropriate.
8. Something is seasonable if it is appropriate to the season.
Something that varies as per season is seasonal. Seasonal The sequence is abbaa. Choice (A)
is more appropriate.
Shear means to cut. Sheer means very thin or transparent. 13. Censor is to suppress; censure is to rebuke. Censured is apt.
Shear is more suitable. Chaotic refers to disturbed. Inchoate refers to a nascent
Silicone refers to a compound derived from the element stage, or early stages. Inchoate is more appropriate.
silicon and is used in many applications. Silicon is the Continually means intermittent, continuously means without
appropriate word. interruptions. Continuously is more suitable.
Straitjacket is a sort of bind to keep violent and unstable Deduct means subtract, take away. Deduce is to conclude
people in check. This is appropriate in the given context. by reason. Deduce is a better way of saying it.
Triumphant Institute of Management Education Pvt. Ltd. (T.I.M.E.) HO: 95B, 2nd Floor, Siddamsetty Complex, Secunderabad – 500 003.
Tel : 040–40088400 Fax : 040–27847334 email : info@time4education.com website : www.time4education.com SM1002114/73
A definite statement is precise and specific, but a definitive Impassible means incapable of emotions or suffering.
statement is final and conclusive on the subject. Definitive Impassable means a road or path that cannot be travelled
is a better fit. on because of bad conditions or because it is blocked.
Impassable is more appropriate.
The sequence is aaaba. Choice (D)
Inequity means unfairness or favoritism. Iniquity is gross
wickedness. Iniquity is more suitable.
14. Disinterested refers to a person who does not have a
selfish motive to get involved. Uninterested refers to a lack The correct order is bbaaa. Choice (D)
of interest. Uninterested is a better word in the context.
Equable means steady, uniform, even-tempered. Equitable 19. Knots is a unit of speed. Knots per hour does not make
means fair division. Equable is more appropriate. sense. Knots is appropriate.
Esoteric means something that is difficult to understand. In the present context let is more apt than leave.
Exoteric means easy to understand. Esoteric is more
Libel is a piece of writing which contains bad and false
suitable.
things about a person Liable means having responsibility
Flaunt is to show off. Flout is to break the rule. Flouted is for something or someone. Liable is more suitable in the
a better word. context.
Inflammable means something that can be set on fire. Loathe means to hate someone or something. Loath
Inflammatory is something that rouses strong emotions. means to be unwilling to do something. Loath is more
Inflammatory is a better word. appropriate.
The sequence is abbba. Choice (C) Meantime is always used with ‘in’, whereas meanwhile can
be used without ‘in’. Hence meanwhile is more apt.
15. Militate means to fight against. Mitigate means to lessen.
The correct order should be babba. Choice (D)
Mitigate is a better word
Proscribed means to prohibit. Prescribed means to order. 20. Minimal means bare minimum. Minimize is to reduce
Prescribed is a better word in the context. something to the least possible level. Minimize fits more
appropriately.
Torturous is something which causes pain or agony.
Tortuous is winding. Tortuous is a better word. Minyan means a minimum of 10 men which was
a prerequisite for Jewish public worship. Minion is a
Venal refers to corrupt. Venial is something that is trivial.
follower or underling. Minion is more appropriate.
Venal is a better word.
Nauseous means the cause which is causing the condition.
Carat is used to denote purity of gold or diamond. Caret is
Nauseated means the condition in which the person is
an inverted ‘V’ used in proof reading. Carat is apt.
about to vomit. Nauseated is appropriate.
The sequence is babab. Choice (B)
Oppress is to inflict hardships on others or on a group.
The word can be applied only to human beings. Repress is
16. Illicit is illegal. Elicit is to get information or reaction. Elicit is
denying one’s own feelings or impulses. Repress is more
apt.
apt.
Indicting is to make formal accusation against someone. To Pancreas refers to a gland that produces insulin. Panacea
indite is to compose or write. Inditing is more apt. is something that is a cure for all ills. Panacea is more
Interpolated means to call for a statement or explanation suitable.
from the member of the Parliament. Interpolate is to insert a The correct order is bbaba. Choice (C)
remark, or throw something into. Interpolated is a better fit.
Irrevocable is something that cannot be changed. Exercise – 6
Irrecoverable is that which cannot be recovered.
Explanatory notes for questions 1 to 40:
Irrecoverable is apt.
Ultimatum is a final warning. Ultimate is something 1. Innuendo is a remark which makes an indirect reference to
happening at the end of a long process. something. Double meaning is a synonym. Choice (D)
The order is babba. Choice (D) 2. Sacrosanct is something extremely valuable. Human life is
also sacred. Choice (A)
17. Effluents refer to chemical waste. Affluent are rich people.
Effluents is appropriate. 3. Hotchpotch means a confused mixture. Jumble is the
Exceptionable means disagreement and dispute. correct word. Choice (C)
Exceptional means extraordinary. Exceptional is
appropriate in the context. 4. Parochialism means to have a narrow outlook or range.
Choice (D)
Further is used in cases of nonphysical dimensions. Farther
is used for distances. Farther is apt. 5. Capricious means prone to sudden changes of moods or
behaviour. Unpredictable means the same. Choice (C)
Feted means honored. Fetid means decayed and stinking.
Feted is more apt. 6. Fractious means difficult to control. Unruly is the correct
Phenomenal means incredible. Phenomenon means a fact choice. Choice (B)
or event which is not fully understood.
7. Penury means extreme poverty. Impoverishment is the
The order is aabba. Choice (A) closest synonym. Choice (A)
18. Former means before or prior. Formal means that which 8. Flout means not following rules etc.; disobey. Spurn means
requires some rules or etiquettes to be followed. Formal is to reject and (D) is not a synonym of flout. Choice (B)
apt.
Hail means to call or wave. Hale and hearty is a phrase 9. Deprecated means to express disapproval. Reprove also
often used for health. Hence hale is appropriate. means to disapprove. Choice (A)

Hoard means to collect something and keep it safe for 10. Presumptuous means behaving with disrespectful
difficult times. Horde means a large group of people. Horde boldness. Overbearingly confident also means the same.
is more appropriate in the context. Choice (D)
Triumphant Institute of Management Education Pvt. Ltd. (T.I.M.E.) HO: 95B, 2nd Floor, Siddamsetty Complex, Secunderabad – 500 003.
Tel : 040–40088400 Fax : 040–27847334 email : info@time4education.com website : www.time4education.com SM1002114/74
11. Euphoria is a feeling of great happiness. Elation also 37. Debunk means reveal something which is false. Expose
means the same. Choice (C) gives the same meaning. Choice (C)

12. Augur means a likely outcome. Portends is the correct 38. Countenance in this context means Appearance, usually of
choice. Choice (A) the face.. Choice (B)

13. Beleaguered means troubled. Harassed is the appropriate 39. Intractable means difficult to deal with Recalcitrant,
word. Choice (B) meaning stubborn, is the appropriate substitute of the word
intractable. Choice (C)
14. Billeted means accommodated. Housed also means the
same. Choice (C) 40. Obliterate means to destroy completely. So the correct
choice is wipe out. Choice (D)
15. Deleterious means causing harm or damage. Harmful is the
appropriate word. Choice (D) Exercise – 7

16. Fatuous means silly and pointless. Idiotically means the Explanatory notes for questions 1 to 50:
same. Choice (B)
1. The events mentioned are economic slowdowns,
17. Esoteric means something which is understood by a few. technological bust and natural catastrophe. What have the
Mysterious is the appropriate choice. Choice (D) companies done in relation to them? They have weathered
(come safely through) not willed (caused by will power),
18. Succumb means to give into. Surrendered nearly means whittled (reduce something by degrees) or wended
the same. Choice (A) (go slowly or by an indirect route). Choice (C)

19. Revanchism means a revengeful desire to re-acquire 2. The companies have faced many obstacles but what has
something lost. The closest synonym is retaliatory. happened to their vision? They have not been deterred
Choice (C) (discouraged) in their vision. They have not been
determined (firm) would suggest that the companies were
20. Wanton means deliberate and unprovoked. Choice (A) weak which is not suggested here. Divested (destroyed or
ruined.), devalued (reduce the worth) are not suitable here.
21. Reticence means not revealing one’s thoughts and Choice (B)
feelings. Diffidence means the same. Choice (D)
3. What has liberalization of the economy presented to these
22. Bedevil means to cause continuous trouble. Torment is the companies, from which they have reaped a rich harvest?
synonym here. Choice (A) Liberalization gives greater opportunities (favourable
circumstances far doing something), it does not present
23. Imprecation means spoken curses. Choice (C) opposition (resistance), oppression (hardships) or output
(the number of things produced). Choice (D)
24. Camouflage means hide. Masked is another word for hide.
Choice (D) 4. The U.S. forces have swept into Baghdad with tremendous
aggression (hostile, violent behaviour or attacking without
25. Rabblerousing means to stir up popular opinion for political provocation). What will be the reaction to it? Naturally there
reasons. Demagoguery is the synonym. Choice (D) has been widespread criticism (indicate the faults of in
a disapproving way) not approval (accept as satisfactory)
26. Jingoism means excessive support for one’s own country. or consolation (comfort received after a loss or
Blustering patriotism means the same. Choice (C) disappointment) or credibility (being believable).
Choice (C)
27. Confiscate means to seize property. Impound means the
same. Choice (A) 5. Disapproval of the tactics ––– how were the tactics?
They were ruthless (having or showing no compassion).
28. Salutary means something advantageous. Beneficial is the They cannot be rustic (characteristic of life in villages),
synonym. Choice (C) or sagacious (having good judgment – since there is
disapproval thus can’t be sagacious) or run-of-the-mill
29. Apologist is a person who argues in defence of something (ordinary). Choice (A)
controversial. Supporter, in this context, is the appropriate
word. Choice (B) 6. The way the U.S. forces behaved has been in contrast to
the behaviour of the British. The latter have used patient
30. Averment means a positive statement. Affirmations means and subtle strategy (plan), not scheme (though scheme can
the same. Choice (A) also mean plan, strategy is the word used in military
context), schedule (timetable) or proposal (suggestion).
31. Taboo means what is not accepted. Prohibited is similar in Choice (B)
meaning. Choice (D)
7. The U.S. wants to bring the battle to a quick end. This has
32. Peripatetic means nomadic. Itinerant means the same. been their consideration. What kind of consideration?
Choice (A) Overweening (showing excessive confidence or pride) and
not outrageous (shockingly bad) or outstanding
33. Consecrate, in this context, means to make or declare that (exceptionally good) or original (existing form the
something is sacred. Sanctifying is the appropriate word. beginning). Choice (C)
Choice (D)
8. We are talking of truck demand and economic health. The
34. Stymie means to prevent or slow down the progress of demand for truck indicates the health of the economy,
something or in other words impede. The closest word here hence it is an indicator (a thing that shows, points out).
is thwarted. Choice (C) Truck demand cannot be an incarnation (a living
embodiment of a deity, spirit or abstract quality) or
35. Garner means to gather. Choice (B)
incorporator (embodies or takes in as part of a whole) or
36. Stigmatize means to condemn or label. Brands is the incinerator (an apparatus for destroying by burning) of
correct synonym. Choice (C) economic health. Choice (D)

Triumphant Institute of Management Education Pvt. Ltd. (T.I.M.E.) HO: 95B, 2nd Floor, Siddamsetty Complex, Secunderabad – 500 003.
Tel : 040–40088400 Fax : 040–27847334 email : info@time4education.com website : www.time4education.com SM1002114/75
9. We have seen that truck demand is an indicator of 20. The Taliban and Al Qaeda are regrouping inside the Pakistan
economic health. Last fiscal, the truck demand has been territory. This can happen only with the connivance (tacit
high (top gear) although the economy has not been robust. approval, secretly allow) of at least a section of the Pakistan
So, both analysts and industry watchers are surprised establishment. There is no connection (link or relationship) or
(baffled). The commercial vehicle front has baffled them consideration (careful thought) or consensus (general
and not bothered (worried), bamboozled (mystified) or agreement). Choice (D)
bailed (released). Choice (A)
21. America has entered the paradise of power. So how is its
10. Since the demand for trucks is directly proportional to the military reach? It defies (challenges) description. It does not
health of the economy, at the height of recession delight (please greatly) or delude (impose a misleading
(economic decline), we would expect truck demand to be belief upon) or delete (remove or erase). Choice (B)
low. Hence the growth is stymied (prevent or hinder the
progress of) and not stubbed (accidentally strike against 22. The clue here is what happened to Iraq. So, America can
something or extinguish by pressing the lighted end against pulverize (defeat utterly) any place. It cannot purchase
something) or stunted (retard the growth or frustrate) or (buy) or purloin (steal) or purge (rid of an unwanted feeling
stunned (astonished, shocked). Choice (B) or condition). Choice (C)
11. For years, the truck demand did not increase. Truck
23. The clue, here is the previous line. America has reached
makers blamed overloading of trucks as the reason.
the limits of its power. So, the seeds that have been sown
Transporters blamed clients who paid measly rates so that
are the seeds of resistance (opposition) and not resignation
operators could not –––– their fleet. What would truck
(acceptance of something undesirable but inevitable) or
operators want to do to their fleet? They would want to
repugnance (intense disgust) or reservation. Choice (A)
revamp (give new and improved form, structure or
appearance to) their fleet. They would not want to revalue
24. ‘Overcome’ suggests that what is referred to are problems.
(value again) or reshuffle (interchange the positions of) or
So, the word is glitches (sudden, usually temporary
reorient (change the focus or direction of). Choice (C)
malfunction) and not glitz (extravagant but superficial
12. It is now agreed that overloading and poor maintenance cut display) or gimmick a (trick or device intended to attract
down the life of the fleet. So, what happens to attention rather than fulfil a useful purpose) or gibberish
replacement? It has to increase, or it is fuelled (sustained (unintelligible or meaningless). Choice (B)
or inflamed) and not fulfilled (achieved or realized
something desired, promised or predicted) or frustrated 25. ‘Designing and fabricating its own satellites’ suggests that
(prevented from progressing) or funded (a sum of money the word is indigenously (native, occurring naturally in
made available). Choice (A) a particular place) and not indiscriminately (without careful
judgment) or inadvertently (not achieved through deliberate
13. The clue lies in the word terror (extreme fear) bonhomie planning) or indiscreetly (revealing what should remain
(good-natured friendliness), benevolence (well-meaning; a secret). Choice (D)
kindly) are both positive words, bondage, though a negative
word does not fit here (Americans are not making the Iraq’s 26. Since the previous launch had failed, it becomes absolutely
slaves) so the word is bombardment (attack continuously imperative (of vital importance) that the launch is perfect
with bombs). Choice (D) this time. It cannot be impersonal (not influenced by
personal feelings), impercipient (failing to perceive
14. The word that follows the blank – humanitarian mission is something) or inexorable (impossible to stop or prevent).
the key. So, it cannot be entrenchment (establish in Choice (A)
trenches) or enslavement (make a slave – which is
negative) or escape (the people of Iraq do not want to leave 27. The word in the blank stands for ‘the company Arian space’,
their country). It is liberation (set free). Choice (C) which word can stand for it? The company can be called
a consortium (an association of several companies) but not
15. The liberation of Iraqi’s is the humanitarian mission of this a conservancy (body concerned with the preservation of
war. Is it really so? There is doubt. Hence, it is purported natural resources) or conservatory (a room with glass roof
(appear to be so, especially falsely). It is not obvious (clear, and walls used as a sun lounge or for growing plants) or
easily perceived – people are not sure), prudent (showing consulate (the place where consuls work). Choice (C)
care and thought for the future – we are talking of the
present), or potential (having the capacity to develop into 28. The clue lies in the opening phrase ____. The fall of Baghdad.
something in future – again it is the present we are It signifies (indicates) something. It does not exhibit (show, or
concerned with). Choice (B) publicly display) or betray “(act treacherously) or expound
(present and explain systematically). Choice (C)
16. In the war against terrorism, what does the U.S want from
Musharraf and Pakistani armed forces? It wants their 29. The weapons of mass destruction – were they really
support or cooperation (work jointly, assist someone) and present in Iraq? They were presumed (suppose that
not contribution (give in order to achieve) or conversion something is the case on the basis of probability, take for
(the process of change) or convention (socially acceptable granted) to be present. They were not presupposed
behaviour). Choice (A) (assumed beforehand or pretended (asserted falsely) or
privatised (make private). Though presupposed and
17. How is the hostility of the Pakistani armed forces towards pretended can fit in the context, presumed is the best.
India? It is compulsive (resulting form or acting on an Choice (B)
irresistible urge) and not confused (bewildered) or feigned
(pretended) or compromising (an agreement reached by 30. We are talking of countries that supported Washington. They
each side making concessions). Choice (C) may be called its cohorts (supporters – used derogatorily)
and not colleagues (persons with whom one works) or
18. How are the links of the Pakistani army with forces entourage (a group of people attending or surrounding
extremism? It is enduring (remain in existence) and not an important person) or collocutors (persons who take part in
engrossing (absorbing), endearing (inspiring love) or a conversation). Since cohorts has a derogatory meaning it
tenuous (weak). Choice (B) fits in with the tone of the passage. Choice (D)

19. What kind of an ally is Pakistan? According to Washington, 31. The search has to be credible (convincing, able to be
Pakistan is a strong ally. Pakistan is not seen as stoic believed) not creditable (deserving praise) or critical
(enduring pain and hardship) or stolid (calm, dependable) (expressing adverse comments) or feasible (possible).
or static (lacking movement). Choice (D) Choice (A)
Triumphant Institute of Management Education Pvt. Ltd. (T.I.M.E.) HO: 95B, 2nd Floor, Siddamsetty Complex, Secunderabad – 500 003.
Tel : 040–40088400 Fax : 040–27847334 email : info@time4education.com website : www.time4education.com SM1002114/76
32. If the weapons of mass destruction are not found, it will be 45. The contrast is between the food grain stocks we have and
interpreted (understood as having a particular meaning) not the occurrence of starvation deaths. The food grain stock is
interrupted (break the continuity of) or interrogated more than four times the optimal (best or most favourable)
(questioned closed, aggressively or formally) or interspersed level. The level of food grain stock cannot be optional (to be
(scattered among other things). Choice (B) chosen but not obligatory) or actual (existing in fact, current
– how can it be four times the current level?) or agreed
33. This war is about seizing Iraq’s oil – this is the way it is (have the same opinion, reach agreement). Choice (B)
widely acknowledged (accepted or admit the truth of).
No one has announced (make a public declaration about 46. The opposition M.P is criticizing the ruling party, and says
a fact) or acquiesced (accept or consent to something the women’s reservation bill is somewhere in the far off
without protest) or conceded (finally admit or agree that future, so it is a mirage (what seems to be but is not there
something is true). Choice (A) in reality, illusory). It is not a possibility (a thing that is
possible) or probability (likely to happen), or potential
34. We are talking about America’s objective (goal or aim) not (having the capacity to develop into something).
aspiration (hope or ambition), or occasion (event) or Choice (C)
opportunity (favourable time). Choice (C)
47. We are talking of a standard to which the government’s
35. America’s aim is not merely oil but it wants to expand its performance can be compared. Hence, the word is
global hegemony (leadership or dominances). Control – benchmark (a standard or point of reference against which
though possible as far as meaning is concerned – does not things may be compared). It is not evaluation
seem an apt word in the context of the passage. (assessment), or assessment (estimation), or beneficiary
Supervision (observe and direct the execution) and (a person who gains benefit). Choice (D)
regulation (rule or directive made and maintained by
an authority) do not fit well. Choice (B) 48. The opposition claims that every single achievement of the
government advertised in the newspaper is false. How is it
36. ‘On the brink of’ (in imminent danger of) suggests that the false? It is demonstrably (clearly apparent or capable of
devastating crisis is yet to come. So, it is potentially (having being logically proved) false. It is not decisively (acting to
the capacity to develop into something in the future) not settle an issue quickly and effectively), or deceptively
positively (certainly) or predictably (occurring in the way (hidden/untrue), or defectively (imperfectly). Choice (A)
expected) or perennially (for ever) devastating crisis.
Choice (D) 49. There was an intelligence failure but what kind of failure? It
was monumental (great in importance). It was not
37. What kind of a disease are we talking about? The clue is in magnanimous (generous), or mortal (liable to die or cause
the names of the diseases – plagues, polio and pneumonia. death – the failure was not mortal, fatal), or multi-lateral
These are not decadent (moral and cultural decline), or (participated in by three or more parties). Choice (D)
infectious (polio is not infectious) or deformational (only
polio is deforming). So, these are debilitating (making weak 50. According to the opposition, the government is not taking
and infirm) diseases. Choice (D) any action but merely indulging in anti-war rhetoric (the art
of effective or persuasive speaking or writing). It is not
38. These diseases were eliminated (completely removed) and eloquence (though eloquence also means fluent or
not merely controlled. Diseases cannot be emancipated persuasive speaking or writing, in the context rhetoric is the
(set free) or emasculated (made weak or less effective). word normally used), or prolixity (tediously lengthy) or
Choice (A) revolution (forcible overthrow of a government or social
order). Choice (B)
39. The modern-day disease not only underlines diverse
conditions such as heart disease but is also a major Exercise – 8
contributor (one who gives in order to help achieve
something) to other disturbances. It is not a controller Explanatory notes for questions 1 to 25:
(having the power to influence behaviour or the course of
events) or contradiction (ideas which are mutually opposed) 1. Choice (A): Correct. This serves as the appropriate
or convolution (a coil or twist). Choice (C) replacement for the given blank.
Choice (B): This option is incorrect because the use of
40. What does the disease do to creativity? It inhibits (hinders, ‘that’ (after ethical code) is wrong.
prevents) creativity. It does not infuse (pervade, fill) or Choice (C): This is incorrect because the preposition ‘in’
injects (introduces) or inhabits (live in or occupy) creativity. should be replaced by ‘by’ in order to make the statement
Choice (B) grammatically correct.
Choice (D): This option is incorrect because the preposition
41. General dissatisfaction is obvious (easily understood, clear) ‘in’ is incorrect, it should be substituted by ‘to’.
in our day to day life. It is not obtrusive (noticeable in an Choice (A)
unwelcome way), offensive (causing hurt), or obstructive
(block, hinder). Choice (D) 2. Choice (A): This option is incorrect because the preposition
‘at’, should be replaced by ‘on’.
42. The Bush administration has decided to pour scorn over Choice (B): This option is wrong because ‘reducing’ does
the U.N. What will be the consequences (result or effect)? not require any preposition after it.
So we have the consequences of ….. not consolidation Choice (C): This option can be eliminated for the same
(make stranger) or consonance (agreement) or consolation reason given for option B.
(comfort). Choice (B) Choice (D): This option makes the sentence grammatically
and structurally correct. Choice (D)
43. America has disabled itself by doing something to the
international system. By what? By repudiating (refusing to 3. Choice (A): This option is incorrect because ‘have’ does not
accept) not renouncing (abandoning or giving up) or agree with the subject ‘summit’.
abandoning or deserting. Choice (C) Choice (B): CORRECT. This option makes the sentence
grammatically and structurally correct.
44. By repudiating the international system, it has disabled Choice (C): This option is incorrect because they are
itself and become isolated (lonely). It has not become victims ‘of’ not ‘to’.
involved (participate in an activity) or intimate (familiar) or Choice (D): This is incorrect because the words ‘Indian Ocean’
invasive (tending to intrude). Choice (A) should be preceded by the article ‘the’. Choice (B)

Triumphant Institute of Management Education Pvt. Ltd. (T.I.M.E.) HO: 95B, 2nd Floor, Siddamsetty Complex, Secunderabad – 500 003.
Tel : 040–40088400 Fax : 040–27847334 email : info@time4education.com website : www.time4education.com SM1002114/77
4. Choice (A): Correct. This is the most suitable option to fill Choice (C): This option is incorrect because the use of the
the blank. article ‘the’, before the proper noun Jericho is incorrect.
Choice (B): This option is incorrect because the verb Choice (D): This option is incorrect because the verb
‘prohibit’ does not agree with the subject ‘legislation’. should be ‘has’ not ‘have’ Choice (A)
Choice (C): A union of states should be preceded by the
article ‘the’, this option is incorrect because of the absence 13. Choice (A): This option is incorrect because the use of the
of the article ‘the’ before the words ‘European Union’. to infinitive along with the ‘ing’ form is erroneous.
Choice (D) is incorrect because it should be subject ‘to’ not Choice (B): Correct. This option fits perfectly in the given
‘with’. Choice (A) blank.
Choice (C): The preposition ‘in’ with violence is incorrect.
5. The prepositions ‘to’, ‘upon’ ‘and’ ‘on’ after the word The appropriate preposition to be used is ‘against’.
integrated in choices (A), (C) and (D) respectively are Choice (D): The preposition ‘at’, with violence is incorrect.
incorrect, The word ‘integrated’ should be followed by ‘with’ Choice (B)
as in choice (B). Choice (B)
14. Choice (A): This option is incorrect because the use of ‘on’
6. Choice (A): The use of ‘whom’ and ‘who’ with reference to after introspection is incorrect.
coastal zone management is incorrect. Choice (B): This option is incorrect because the use of ‘for’
Choice (B): The use of ‘who’ with reference to coastal zone after introspection is incorrect.
management is incorrect. Hence choices (A) and (B) are Choice (C): This option is incorrect because the preposition
incorrect. should be ‘over’ not ‘upon’ sharing power.
Choice (C): This option is incorrect because the use of ‘to’ Choice (D): Correct. This option makes the sentence
with the verb + ing form is incorrect (transform + ing). grammatically and structurally correct. Choice (D)
Choice (D): Correct. This option is most suitable in the
given blank. Choice (D) 15. Choice (A): This option is incorrect because ‘east coast’
should be preceded by the article ‘the’.
7. Choice (A): This option is incorrect because the verb ‘drive’ Choice (B): Correct. This option makes the sentence
should be in the past participle (driven) form. grammatically and structurally correct.
Choice (B): Correct. This option is most appropriate in the Choice (C): This option is incorrect because the verb have
given blank. does not agree with the subject (the tsunami onslaught).
Choice (C): This option is incorrect because the preposition Choice (D): This option is incorrect because the industry is
should be ‘by’, not ‘with’. ‘in’ A.P not ‘to’ A.P. Choice (B)
Choice (D): This option is incorrect because it should be
seem not ‘seems’ Choice (B) 16. Choice (A): This option is incorrect because the word
‘under developed’, should be preceded by ‘an’.
8. Choice (A): This option is incorrect because the word ‘most’ Choice (B): This option is incorrect because the word
should be preceded by the definite article ‘the’. ‘seem’ does not agree with the subject ‘image makeover’.
Choice (B): This option is incorrect because the verb ‘have’ is Choice (C): Correct. This is the appropriate option to fill the
in the plural form. The word ‘water’ is a singular uncountable blank.
noun therefore it should be followed by the singular verb Choice (D): This option is incorrect because ‘to having’ is
‘has’. wrong. Choice (C)
Choice (C): Correct. This is the correct option to fill the blank.
Choice (D): This option can be eliminated for the same 17. Choice (A): This option is incorrect because the word
reason given for option B. Choice (C) descended should be followed by the preposition ‘from’.
Choice (B): This option is incorrect because proper nouns,
9. Choice (A): Correct. This option makes the sentence Mauritius here, should not be preceded by ‘the’.
grammatically and structurally correct. Choice (C): This option is incorrect because the use of the
Choice (B): This is incorrect because the comparison is preposition ‘to’, after the word tracing’, is wrong.
made between two countries and hence the preposition Choice (D): This is the correct option to fill the blank.
‘between’ is appropriate. Choice (D)
Choice (C): This option is incorrect because the preposition
‘ from’ should be replaced by ‘since’ because a point of 18. Choice (A): Correct. This option is grammatically and
time is being referred to. structurally correct.
Choice (D): This option is incorrect because the verb Choice (B): This option is incorrect because the word
‘surround’, should be in the ‘ing’ form. Choice (A) ‘dozen’ should be preceded by the article ‘a’.
Choice (C): This option is incorrect because the word
10. Choice (A): This option is incorrect because it should be Indian Ocean, should be preceded by the article ‘the’.
encompasses. Choice (D): This option is incorrect because the use of the
Choice (B): This option is incorrect because ‘so as’ should preposition ‘during’ is incorrect. It should be by ‘since’.
be replaced by the conjunction ‘such – as’. Choice (A)
Choice (C): This option is incorrect because the word ‘who’
19. Choice (A): This option is incorrect because the word
should be replaced by ‘which’.
‘hundred’, should be preceded by the article ‘a’.
Choice (D): Correct. This option is apt to fill the blank.
Choice (B): This option is incorrect because the word long
Choice (D)
should be preceded by the article ‘a’.
Choice (C): This option is most appropriate in the given
11. Choice (A): This option is incorrect because the use of the
blank.
‘that’ after the word ‘show’ is incorrect.
Choice (D): This option is erroneous because the verb
Choice (B): Correct. This is the correct option to fill the
‘have’ does not agree with the subject ‘nation’ which is
blank.
singular. Choice (C)
Choice (C): This option is incorrect because the use of the
article ‘the’, before the word ‘little’ is wrong. 20. Choice (A): This option cannot be the right answer because
Choice (D): This option is erroneous because the the word suffering should be followed by ‘from not ‘with’.
preposition should be ‘in’ not ‘on’. Choice (B) Choice (B): Correct. This option is grammatically and
structurally correct.
12. Choice (A): Correct. This is the most suitable option to fill Choices (C) and (D) are incorrect because it is availability
the blank. ‘to’ not ‘for’. The word ‘what’ in choice (C) should be
Choice (B): This option is incorrect because ‘Jordan river’ replaced by ‘which’ and in choice (D) the word ‘most’
should be preceded by the article ‘the’. should be preceded by the article ‘the’. Choice (B)
Triumphant Institute of Management Education Pvt. Ltd. (T.I.M.E.) HO: 95B, 2nd Floor, Siddamsetty Complex, Secunderabad – 500 003.
Tel : 040–40088400 Fax : 040–27847334 email : info@time4education.com website : www.time4education.com SM1002114/78
21. Choice (A): Correct. This option is appropriate in the given 4. Choice (A): Correct. This option is grammatically correct.
blank. Choice (B): The usage of ‘is increasingly’ is incorrect as the
Choice (B): This option is incorrect because of the use of subject is in plural i.e., applications, therefore ‘are’ should be
the present continuous tense (is having). used.
Choice (C): This option is incorrect because the word Choice (C): The usage of ‘but’ is incorrect as ‘but’ always
‘eminent’ should be preceded by the article ‘an’. shows contradiction between statements.
Choice (D): This option can be eliminated for the same Choice (D): This option which contains errors seen in both
reason given for C. Choice (A) (C) and (D) is incorrect. Choice (A)

22. Choice (A): This option is incorrect because the word 5. In choice (A) and (B) the usage of the preposition ‘on the
premised which precedes the blank should be followed by international market’ is incorrect and should be ‘in the
the preposition ‘on’. international market’. Further, ‘will be vetting’ is incorrect in
Choice (B): This option is incorrect because US should be (A). In (D), the verb ‘been’ without the verb has is incorrect.
preceded by the article ‘the’. Hence choice (C) is correct. Choice (C)
Choice (C): Correct. This option is appropriate in the given
blank. 6. Choice (A): The definite article ‘the’ is used before names
Choice (D): This option can be eliminated for the same of things unique of their kind. ‘The’ is to be used before
reason given for A. Choice (C) army, police and paramilitary essentially. Hence, option A is
incorrect.
23. Choice (A): This option is incorrect because oceans should Choice (B): The word ‘police’ should be preceded by the
be followed by ‘have’. definite article.
Choice (B): Correct. This option makes the sentence Choice (C): Correct. This option is grammatically correct.
grammatically and structurally correct. Choice (D): This option is incorrect because the comma
Choice (C): This option is incorrect because the word major before ‘since’ changes the meaning of the sentence and
should be preceded by the article ‘a’. also ‘at their’ will is not correct. Choice (C)
Choice (D): This option is incorrect because ‘source’ should
be followed by the preposition ‘of’. Choice (B) 7. Choice (A): The absence of comma after today and no one
makes the sentence incorrect.
24. Choice (A): Correct. This option makes the sentence Choice (B): Correct. This option is grammatically correct.
correct in terms of grammar and usage. Choice (C): The expression ‘comes’ through a ‘price’ is
Choice (B): This option is incorrect because the use of the incorrect.
word ‘affect’ is incorrect. Choice (D): The expression ‘come at a prize’ is incorrect.
Choice (C): This option is incorrect because ‘his’ should be Choice (B)
replaced by ‘one’s’ since the sentence has ‘ones’ fat’s.
Choice (D): This option is erroneous because the verb 8. Choice (A): Correct. This option is grammatically correct.
‘was’ does not agree with the subjects ‘gemstones’, which Choice (B): The starter in B ‘whereas’ suggests that there is
is plural. Choice (A) a comparison, which is incorrect.
Choice (C): In C, ‘while in India suggests that only while
25. Choice (A): This option is erroneous because the ‘to’ infinitive, living in India. Further, the usage of ‘involving’ suggests
cannot be followed by the ‘ing’ form (to strengthening). that the multi-disciplinary approach is for the psychologists
Choice (B): This option is incorrect because we are etc.
accountable ‘to’ god not ‘for’ god. The change in preposition Choice (D): “That is involving” should be replaced by “which
changes the meaning (eg. We are accountable to the involves”. Choice (A)
organisation but we are accountable for our mistakes).
Choice (C): This option is incorrect because the verb 9. Choice (A): The usage of preposition ‘with’ regarding visas
‘generate’ is wrong. is incorrect.
Choice (D): Correct. This option makes the sentence Choice (B): Similarly, ‘by visas’ in (B) is incorrect. In (C) the
grammatically and structurally correct. Choice (D) structure ‘say sometimes’ is not correct, also with reference
to ‘countries’ the relative pronoun should be ‘who’ and not
Exercise – 9 ‘which’ because countries is personified here.
Choice (D): Correct. This option is grammatically correct.
Explanatory notes for questions 1 to 20: Choice (D)
1. Choice (A): The inference one draws from the sentence 10. Choice (A): ‘Politics’ as mentioned in the sentences should
‘the crowd included……….. marching from’ is that only be followed by ‘has’ and not ‘have’.
those people were marching which is incorrect. Choice (B): Correct. This option is grammatically correct.
Choice (B): The usage of ‘had marched’ and ‘when’ can’t Choice (C): Besides containing the error mentioned in A,
be used because of different time frames. ‘inflicted with’ is incorrect.
Choice (C): Correct. This option is grammatically correct. Choice (D): The expression ‘independence upon’ is
Choice (D): The words ‘having dressed’ are incorrect. incorrect. Choice (B)
Choice (C)
11. Choice (A): The relative pronoun ‘who’ is incorrect with
2. Choice (A): The conjunction ‘but’ is incorrect as both the regard to countries.
ideas expressed are the same ‘and’ is the correct Choice (B): ‘India is only Russia’s partner’ suggests that
conjunction. India has no other partner so (B) is incorrect.
Choice (B): This option begins with ‘having renewed’ which Choice (C): ‘India is the only Russia’s partner’ suggests
is the incorrect verb form. one and only Russia which is understood and doesn’t make
Choice (C): This option begins with ‘renewing’ which is good sense.
incorrect. Choice (D): Correct. This option is grammatically correct.
Choice (D): Correct. This option is grammatically correct. Choice (D)
Choice (D)
12. Choice (A): Stylistically an event comes first and time frame
3. Choice (A): Correct. This option is grammatically correct. later, hence (A) is incorrect.
Choice (B): The phrase ‘in better using their skill’ is incorrect. Choice (B): Correct. This option is grammatically correct.
Choice (C): The phrase ‘in better usage in their skills’ is Choice (C): ‘Majority of communication’ takes the verb ‘is’
incorrect. since it is an uncountable noun. Hence ‘are’ is incorrect in
Choice (D): The implication is that ‘their skills are used to (C) and (D).
harness their intellectual capital’ is not correct. Hence Choice (D): Further, ‘communications’ in (D) is not correct.
choice (A). Choice (A) Choice (B)
Triumphant Institute of Management Education Pvt. Ltd. (T.I.M.E.) HO: 95B, 2nd Floor, Siddamsetty Complex, Secunderabad – 500 003.
Tel : 040–40088400 Fax : 040–27847334 email : info@time4education.com website : www.time4education.com SM1002114/79
13. Choice (A): Correct. This option is grammatically correct. 2. The error lies in statements b and c. In b the adverb
Choice (B): The time frame ‘no longer’ or ‘any longer’ ‘thoroughly’ should come at the end – pampered him
usually should be placed at the beginning of an idea or at thoroughly’. In c the preposition ‘for’ is wrongly used. It
the end of an idea. With this in new we can rule out (B) and should be replaced by the preposition ‘to’. Choice (D)
(D). Further in ‘B’ the conjunction ‘neither’ should be
followed by ‘nor’. 3. Statements b and c are erroneous. The preposition ‘in’
Choice (C): ‘The rest of the word’ is considered an should be replaced by ‘on’ in statement b. Statement c is
uncountable whole noun and hence only ‘much’ can be erroneous because the preposition ‘among’ should be
used here, ‘more’ is incorrect. replaced by ‘between’ because the issue is related to two
Choice (D): Besides containing the error mentioned in B, persons. Choice (C)
this option is incorrect because the word “elite” should be
preceded by “an” and not ”a” Choice (A) 4. Statement c is erroneous because the preposition ‘of’
should be replaced by ‘for’. Choice (D)
14. Choice (A): The ordering of the words is incorrect, ‘any
longer’ has to come after the complete idea expressed. 5. Statements b and c are erroneous. In statement b the word
Choice (B): Correct. This option is grammatically correct. ‘would’ should be replaced by the word ‘should’. In
Choice (C): ‘Those’ is missing before who and so is not statement c the preposition ‘in’ should be replaced by ‘with’
correct. (jumped with joy). Choice (D)
Choice (D): The ordering of words in (D) is totally incorrect
Choice (B) 6. Statements a and c are erroneous. In statement a the
preposition ‘for’ should be replaced by ‘to’. In statement c the
15. Choice (A): The word ‘harmonisation’ takes the preposition preposition ‘to’ is incorrect in the second place. It should be
with and not ‘to’. Further, the relative pronoun ‘that’ is deleted – ‘to make yourself look good’. Choice (D)
incorrect.
Choice (B): The use of the relative pronoun “that” is 7. The error lies in statement d. The preposition ‘at’ is
incorrect. incorrect it should be replaced by ‘through’. Choice (D)
Choice (C): Correct. This option is grammatically correct.
Choice (D): The word ‘harmonisation’ takes the preposition 8. The error lies in statements b and c. In statement b the
with and not ‘to’. Choice (C) article ‘a’ should be replaced by ‘an’. In statement c the
preposition ‘in’ should be replaced by ‘to’. Choice (D)
16. Choice (A): The preposition ‘to’ between the words
considering and passing is incorrect. 9. Statements a and b are erroneous. In statement a, the
Choice (B): ‘Corporation in’ is incorrect, it should be article ‘a’ is incorrect, it should be replaced by ‘an’. In
‘corporations form’. statement b, the error lies in the apostrophe which should
Choice (C): Correct. This option is grammatically correct. be deleted. Choice (C)
Choice (D): The preposition ‘in’ between the words
considering and passing is incorrect. Choice (C) 10. The error lies in statement b. The use of the word ‘who’ is
incorrect in Choice (B), it should be replaced by ‘which’
17. Choice (A): The verb ‘exists’ is incorrect as the ‘Vedas’ are because it refers to English which is inanimate.
plural and hence should be ‘exist’. Choice (A)
Choice (B): The same error features in (B) which says ‘is
not written down’ but should be ‘are not written down’. 11. Statements b and d are erroneous. In statement b the ‘to’
Choice (C): ‘Between’ suggests something between two. infinitive along with the ing form (going) is incorrect.
But ‘holy edicts’ are many and hence (C) is incorrect. Hence the to infinitive should be deleted. In statement d the
Choice (D): Correct. This option is grammatically correct. preposition ‘of’ should be replaced by ‘from’. Choice (C)
Choice (D)
12. The errors lie in statements a, b and c. In statement a, ‘of’
18. Usually the word ‘attempt’ takes the preposition ‘to’ hence should be replaced by ‘off’. In statement b the word ‘so’
choice (D) is incorrect. But in choice (A) the ‘ing’ form of the after the word ‘tired’ is incorrect. The correction should be
verb after ‘to’ is incorrect. For people the relative pronoun ‘so tired that’. In statement c the proposition ‘from’ should
to be used is ‘who’ and not ‘that’ so ‘human beings’ takes be replaced by the preposition ‘by’. Choice (B)
the pronoun ‘who’ hence choice (B) is incorrect (C) is the
correct choice. Choice (C) 13. Statements b and c are erroneous. In statement b the
preposition ‘at’ should be replaced by ‘into’. In statement c
19. Rapid deforestation etc will ‘result in’ radiation hazards and the preposition ‘in’, is incorrect, it should be replaced by
global warming. The words ‘concluding in’ ‘resulting to’ and ‘on’. Choice (D)
‘resulting from’ in choice ‘(A), (C) and (D)’ are incorrect.
Choice (B) 14. The mistakes are in statements b and c. Statement b is
wrong because the word ‘lot’ should be preceded by the
20. In (B) the relative pronoun ‘who’ is incorrect as the subject article ‘a’. Statement c becomes grammatically correct
previously is governance. Further, the usage of ‘will’ speaks when ‘of’ is deleted. Choice (A)
of something definite but in this case nothing definite can
be predicted with regard to the result of proactive 15. Statements a, b and c are incorrect. Statement a is
governance. In which case (B), (C) and (D) are incorrect. In incorrect because of the preposition ‘with’ should be
(A) ‘that can’ points out to a possibility which is correct. replaced by ‘to’. Statement b is incorrect because ‘since’
Choice (A) should be replaced by ‘for’, because a period of time is
being referred to. Choice (D)
Exercise – 10
16. Statements b and d are erroneous. In statement b the
preposition ‘on’ should be replaced by ‘in’. In statement d,
Explanatory notes for questions 1 to 25:
the usage of ‘much’ is incorrect, it should be replaced by
‘many’ or ‘most’. Choice (D)
1. The errors lie in statements a, c and d. a is wrong because
of the absence of the article ‘the’ before the word ‘key’. 17. Statement b is incorrect because the preposition ‘for’
Statement d is erroneous because of the word ‘including’, should be replaced by ‘to’. Statement d is incorrect
in this context the verb ‘include’ should be used without the because of the absence of the article ‘a’.
‘ing’ form. In c, the future tense ‘will’ experience is The word ‘complaint’ should be preceded by the article ‘a’.
erroneous. Choice (C) Choice (C)
Triumphant Institute of Management Education Pvt. Ltd. (T.I.M.E.) HO: 95B, 2nd Floor, Siddamsetty Complex, Secunderabad – 500 003.
Tel : 040–40088400 Fax : 040–27847334 email : info@time4education.com website : www.time4education.com SM1002114/80
18. Statement b is incorrect because the word ‘with’ should be 7. In statement b, ‘aspect in his life ‘ is faulty, it should be
replaced by the preposition ‘in’. Statement d is incorrect ‘aspect of his life’. In statement c the indefinite article ‘an’ is
because the word ‘more’ should be replaced by ‘most’ or missing before atheist. In statement d it should be
‘much’. Choice (A) ‘references to’ not ‘of’. In statement e scattered should be
followed by ‘across ‘ not ‘along’. Hence only statement a is
19. Statements c and d are erroneous. Statement c is incorrect correct. Choice (D)
because the word ‘boost’ should be in the plural form
(boosts) because the verb should agree with the subject 8. Statement a is faulty as ‘so’ should be replaced by ‘as’. ‘So’
(systematic approach). In statement d the preposition ‘at’ should be followed by ‘that’. In statement e the conjunction
should be replaced by ‘in’, in order to make the sentence ‘but ‘ should replace ‘and’ since the ideas are contrasting.
grammatically correct. Choice (D) Choice (A)
20. The error lies in statement d. where the future tense (will) is 9. In statement b, ‘beginning’ should be spelt correctly. In
wrong. The tense (simple present) should remain statement e noteably is spelt errorneously. It should be
consistent …. that leads to over eating. Choice (C) notably. Choice (B)
21. Statement c is incorrect because of the wrong tense. The 10. In statement d, ‘could’ is in the past form. Since the
word ‘is’ should be replaced by ‘has’. Statement d becomes sentence is in the present, it should be ‘can’. In e, atoms is
grammatically correct if the preposition ‘in’ is replaced by a countable noun hence ‘Few atoms’. Choice (D)
‘at. Choice (C)
11. In statement b, ‘as’ is missing, hence, faulty. The
22. Statements c and d are erroneous. In statement c, coustruction is ‘as…….as’. In statement d ‘phenomenon’ is
‘flourishes’ (not flourish) would agree with ‘creativity. singular whereas there a plural ‘ phenomena’ is required.
Statement d is incorrect because the verb ‘includes’ does Hence a, c and e are corret. Choice (C)
not agree with the subject ‘conditions’, hence the verb
should be ‘include’. Choice (B) 12. In statement b, it should be extra skills. In statement e
"branch out into other areas is correct. “Hence a, c and d
23. Statements a and c are erroneous. Statement a is are correct sentences. Choice (D)
erroneous because of the absence of the definite article
‘the’ before ‘Ganga’. Statement c is incorrect because the 13. In statement a, ‘barely’ is misplaced. ‘Barely a glance’ is
word ‘city’ should be replaced by ‘cities’. Choice (A) correct.
In statement e ‘a’ before strong arts culture is missing.
24. The error lies in statement b. in which ‘about’ should be Hence only b, c and d are correct sentences.
replaced by ‘for’. Choice (C) Choice (B)
25. Statement a and d are erroneous. Statement a should be 14. In statement a, a comma after here is required In statement
‘Mini and I….’. Statement c is erroneous. The past perfect c instead of ‘much’ – ‘more’ should be used, i.e. more
tense ‘had reached’ is incorrect. The past tense ‘reached’ is freedom. In statement d instead of exploring, the infinitive
adequate. Statement d becomes grammatically correct if ‘to explore’ should be used.
the word ‘cold’ is preceded by the article ‘a’. Choice (D) Hence b and e are correct. Choice (C)
Exercise – 11 15. In statement a, instead of farm it should be farming – which
is the industy. In statement c instead of the simple past
Explanatory notes for questions 1 to 25:
tense, present perfect tense should be used ie. ‘ has
1. In statement b, the verb is faulty , since its subject is delivered’. In e ‘now have acess’ is correct.
‘AlQaeda and its philosophy’ the verb should be plural ie ‘Have’ should not be used in the progressive tense.
‘are’. In statement d there is no aspostrophe for Choice (A)
orgainisations. Since it refers to belonging ‘s is to be
16. In statement a ,since the sentence refers to a period of time
inserted . Moreover ‘large-scale’ is used as an adjective,
‘ present perfect tence’ should be used.’I have seen ‘is
hence it should be hyphenated. In statement e to maintain
correct. In statement d ‘could’ should be replaced by ‘can’.
parallelism in the structure ‘small’ should be in the
Hence b, c and e are correct. Choice (D)
comparative form i.e. smaller. Hence only statements a and
c are correct. Choice (C) 17. In statement b, ‘mathematic’ should be mathematical as it
is an adjective modifying fluency. In statement c ‘an’ before
2. In statement c, ‘effort’ must be plural ‘efforts’ is correct as it
‘essential’ is missing. In statement e ‘as well as ‘ should be
involves most goverments. In statement e ‘has is faulty, it
replaced by and. “Both ……. and” are a pair of
should be ‘have’ because we are talking of busting, tracking
co-ordinating conjunctions, one must follow the other.
and hunting activities. Choice (D)
Choice (C)
3. In statement a, since the adverb ‘predictably’ modifies the
18. In statements a and d ‘capacty of ‘ is incorrect. it should be
whole sentence it should be placed right in the beginning.
‘capacity for’. In statement c it should be ‘sets us’ instead of
In statement e "was evaporating” is faulty as the sentence
‘set us’. In statement b conscious is spelt incorrectly. Only
has to be in the simple past "evaporated” is correct.
statement e is correct. Choice (B)
Choice (D)
19. In statement a, the adverb’ absolutely’ is required, not
4. In statement c, the preposition ‘in’ should be replaced by
‘absolute’. In statement d the past form of will i.e. would is
‘at’. In statement d ‘long-term’ should be hyphenated as it is
appropirate.In statement e ‘carry the day’ is the correct
used as an adjective. In statement e ‘to wean’ should be
phrase not lift the day.Hence only b and c are correct.
followed by a reflexive pronoun ‘itself’ Choice (A)
Choice (D)
5. In statement a, the comparative of ‘easy’ ‘ easier’ should be
20. In statements a and c the apostrophe, which denotes
used, since it involves a comparison. In statement d ‘only ‘
posession, is not required. Only s without aspostrophe is
should be palced in the beginning of the sentence. Hence
added. In statement d the spelling of ‘microorganisms’ is in
only b, c and e are correct. Choice (B)
correct. Hence only b and e are correct. Choice (D)
6. Statement a is faulty as ‘one of the many’ renders ‘facet ‘
erroneous. It should be facets. Statement d is faulty as 21. In statement a, after ‘are’ a comma is missing. In statement
‘one’ in the previous part of the sentence, should be d since it is a positive sentence ‘some’ should be used
followed by ‘ one’ not ‘you’. Hence only b, c and e are instead of ‘any’. In statement e ‘in the sructure ‘ is correct.
correct. Choice (C) The preposition into denotes motion. Choice (C)

Triumphant Institute of Management Education Pvt. Ltd. (T.I.M.E.) HO: 95B, 2nd Floor, Siddamsetty Complex, Secunderabad – 500 003.
Tel : 040–40088400 Fax : 040–27847334 email : info@time4education.com website : www.time4education.com SM1002114/81
22. In statement a, ‘wheeled robots’ is correct. To maintain If we look at the choices (B) and (C), a is followed by c or a
parallelism in the sentence ‘defusing’ should be used to go is followed by b. Now we have to decide between b and c
with ‘checking’. Hence statement c is faulty. Hence only b, to follow a. b says that it was officially recognised in 1947.
d and e are correct. Choice (A) c gives the information when it actually began to focus on
children and ceased to be a general one. Hence statement
23. In statement a, ‘Times’ need not have an apostrophe. In c is a continuation of the idea expressed in a. Thus a
statement e ‘apparents is wrongly spelt. Hence b, c and d followed by c is the most appropriate combination. Further
are correct sentences. Choice (B) c is followed by b in which we find a reference to when it
was officially recognised. i.e. acb is the logical order of the
24. In statement a, ‘genarally’ an adverb should be placed statements which is found in (B). Choice (B)
before the verb ‘thought’ as it modifies it. In statement c the
indefinite article ‘a’ is missing before ‘bubble’. In statement 4. Statement 1 says ancient remedies for physical and mental
d ‘exponential’ is wrongly spelt. Hence b and e are correct well-being are once again being accepted. In statement c
sentences. Choice (D) we find a mention of the age-old remedies for a number of
problems. Thus c is the most appropriate statement to
25. In statement a, the coordinating conjunction ‘not only’ is follow 1. Choices (C) and (D) begin with c. Now we have to
misplaced. It should be placed after determined. In decide between d or a to follow c. Statement d says, still
statement e ‘patterns’ is correct. Hence only b, c and d are human beings are not able to overcome some illness.
correct sentences. Choice (C) Statement a says apart from concentrating on the ancient
methods of remedy science has been taking strides in all
Exercise – 12 areas. Thus, a can be a better choice to precede d.
Thus, ca is a better combination which is further followed
Explanatory notes for questions 1 to 15: by d. The combination cad is found in (D). Choice (D)

1. Statement 1 says children and adolescents at all levels 5. Statement 1 says to carry out our day to day activities
should be made to learn critical health and skills. b is the minimum amount of endurance and flexibility in our bodies
most appropriate statement to follow 1. The words ‘the set is needed. If we look at the statements, d is the most
of skills’ in b is a continuation of the idea expressed in 1. appropriate one to follow 1. d emphasizes on the fact that
It also defines ‘life skills’ expressed in 1. Choices (A) and our physical activities are adversely affected, which is a
(B) begin with b. Now we have to decide between c and continuation of the idea expressed in 1. Choices (A) and
a to follow b. Statement b defines what constitutes life (D) begin with d. Now we have to decide, if d followed by c
skills. Statement c says no text book can solve these is a better choice or d followed by a. Statement a says
issues. Here the word ‘issues’ refers to the difficulties there are some unforeseen emergencies in store for all of
related to psychological competence. The difficulties are us. It has no logical relation with d. Hence it cannot follow.
mentioned in statement c, which should precede statement d. The words “the realization of this ….” refers to the reason
a. Hence ‘bca’ is a better combination. Hence choice (B) for our abilities to be adversely affected. Hence c is the
can be ruled out. If we look at choices (A) and (C) bc and better statement to follow d. Further in b we find emphasis
ad go together. Now we have to decide between bc and cb. on the current ways of life. Thus, dcb is the logical order
c states the difficulties encountered by students and b which is further followed by a. Thus (A) is the answer
defines the life skills. Thus b should precede c. choice. Choice (A)
Hence choice (C) is also ruled out. Thus statement c gives
the reason for the death of children as stated in 1. If we 6. Statement 1 states that revenge is nothing but paying back.
look at choices (B) and (C) ca and cb are combinations. i.e. it defines the term revenge. If we look at the
Now we must decide between a and b to follow c. statements, we find cb go together and ‘ad’ go together.
Statement a gives the names of the diseases mentioned in These combinations are found in choices (A) and (B). Now
c. If we look at statement b it gives the statistical we have to decide between a and c to follow statement 1.
information related to premature deaths, which is more, Statement a says that revenge assumes many forms. If we
appropriately the concluding statement. Hence ca is the look at statement c it is understood that the word ‘it’ refers
better choice to follow 1 and b a concluding statement. to revenge. The idea expressed in statement 1 finds a
Choice (A) continuation in statement c. Thus c is the more appropriate
statement to follow 1. Choice (A) can be ruled out. Choices
2. Statement 1 says that the common house sparrow has now (B) and (D) have c as the beginning statement. Now we
become scarce. Statement c is the most appropriate choice have to decide between b and a to follow c. Statement c
to follow 1. c says miners carried Canaries into mines. says the form of revenge depends on who takes it.
[canary: a bright yellow finch with a melodious song, Statement a says since human nature varies, revenge also
popular as a cage bird]. Further in statement d it is said that takes many forms. If statement a follows c immediately, the
if canaries are distressed, it is understood that there are idea is not completely expressed. Thus choice (D) can be
poisonous gases. Thus d is the appropriate statement to ruled out. If we look at b, it says the form of revenge
follow c. Statements cd stand as examples to the problem reflects the personality of the person. Statement b is a
expressed in statement 1. The combination cd is found in continuation of the idea expressed in c. Hence cb is a
(A), (B) and (D). If we look at choice (A) statement d may better combination. Further a says revenge too assumes
precede 6. d says if canaries show signs of distress, it many forms depending on human nature the words “the
indicates that there are poisonous gases. 6 says the personality of the person” in b find a relation with the words
number of bird species help in estimating the quality of the “human nature” in a. Further statement d is the most
environment. Thus d is not logically related to 6. Thus appropriate statement to precede 6. Statement d says very
choice (A) is ruled out. If we look at the statements, A is the few of them create an impact. Statement 6 says they
most appropriate statement to precede 6. a says declining remain inactive and lack creativity. Thus statements d
bird population indicates increase in air pollution. Further in should precede 6. Hence the most appropriate choice is (B)
6, it is said in countries like Britain, quality of environment is Choice (B)
calculated based on the number of bird species. Since c is
the most appropriate statement to follow 1 and a is the 7. Statement 1 says ocean is the area that still remains
most appropriate concluding statement. Choice (B) unexplored. If we look at the choices a, c or d may be the
appropriate statements to follow 1. Statement a says India
3. Statement 1 says Bethnal Green Museum is a tribute to has a vast coastline. It has no relation with the idea
children. If we look at the statements, a is the most expressed in 1. Hence 1 can be ruled out. Statement c
appropriate statement to follow 1. a tells us about how the says only a small fraction of the large resources have been
museum began with works of art and about its location. tapped in our country. It has no relation with the idea

Triumphant Institute of Management Education Pvt. Ltd. (T.I.M.E.) HO: 95B, 2nd Floor, Siddamsetty Complex, Secunderabad – 500 003.
Tel : 040–40088400 Fax : 040–27847334 email : info@time4education.com website : www.time4education.com SM1002114/82
expressed in 1. Hence choice (B) can be ruled out. Now we warming” in b is a continuation of the idea expressed in 1. If
have to decide between da and db. Statement d says we look at choices (C) and (D) we have to decide between
oceans are virgin territories whose energies are yet to be b followed by d or b followed by a. b says what is global
tapped. It finds a continuation of the idea expressed in 1. warming. d states about earth’s geological history but in a it
Further if we look at statement a it emphasizes on the vast is said about the origin of the earth. Thus statement a
coast line of India. There is no direct relation between the should precede statement d. Thus ba go together and
ideas expressed in d and a. Hence choice (C) can be ruled further d follows. This combination is found in choice (D).
out. Statement b says the oceans of the world if exploited Choice (D)
have a wealth of energy. This is a continuation of the idea
expressed in d. Hence db is the better combination. Further 12. Statement 1 defines summer in the past. If we look at the
b is followed by ac. a is the example to the generalized idea choices b, c or d may follow statement 1. If we look at
expressed in b. It states about vast Indian coastline and c statement c the words “Even others …” do not find any
says that only a fraction of the oceans have been tapped in relevance with the idea expressed in 1. Hence choice (C)
our country. Thus ac are interrelated. Choice (D) can be ruled out. Statement 1 emphasizes on the past. If
we take statement b it states about something in the
8. Statement 1 says people can set goals, but many people present. It cannot follow statement 1 immediately. Thus
get deviated from their goals. If we look at the choices b, c choice (C) is ruled out. Now between choices (A) and (B)
or d may be the appropriate statements to follow 1. we must decide the appropriate choice. In both the choices
Statement c says top achievers do not get deviated. we find that db go together. We have to decide between bc
Statement c cannot follow 1 immediately because without and ba. b is about people in teaching profession. a says
discussing the reason for getting side tracked we cannot summer is a preparation period. These two statements do
use the term ‘top achievers’. Statements b and d state why not have a logical relevance. c is about people in other
people get side tracked and what are the things that side professions i.e. it says those people of other professions
track people. Hence statements b and d should precede c. also spend a bit of their time with children. Hence c is a
Thus choice (C) is ruled out. Now we have to decide continuation of the idea expressed in b. From the
between b and d to follow 1. b tells about why one is side explanation it is understood that dbc are in logical order
tracked. d says what type of things generally side track which is found in (A). Choice (A)
people which is an extension of the idea expressed in b.
Thus b followed by d is the better choice. Hence choice (D) 13. Statement 1 states the Indian mythological idea on the
is ruled out. The choices (A) and (B) have bd combination. moon. From the choices it is understood that b, d or c may
Now we must decide between dc and da [in choices (A) follow statement 1. If we take statement c it says the moon
and (B)]. d says that the things that side track people are is a fascination for all the people in the world. But the idea
good things but not the best. Further if we look at statement of “Indian mythology” in 1 and of “fascination” in c are not
a the word “They” does not give us clarity about whom it related. Hence choice (A) can be ruled out. If we look at
refers to. Thus da is not the appropriate combination. If we statement d we find that the moon is personified. It cannot
look at statement c we find a continuation of the idea follow 1 immediately. The words “appealing” and “cooling”
expressed in d. Hence from the explanation we understand used in 1 find a continuation in b. The words ‘pearls’ and
that bdc is the logical relation which is found in choice (A). ‘white lilies’ in b describe the idea expressed in 1. Hence
Choice (A) statement b is more appropriate to follow 1. If we look at
choices (B) and (C) we must decide between ba or bd. If
9. Statement 1 says unbridled anger among Indians has we look at the statements b and d we find that they go
become a world phenomenon. From the choices we together. In both the statements the moon is personified
understand that statements b, c, d or a may follow and a romantic description is presented. Thus choice (C)
statement 1. Statement d says “It gives rise to ….”. It does can be ruled out. bd is further followed by a which says why
not give a clear idea of what does “It” mean. Hence d the moon is called Chandamama and then by c which says
cannot follow 1. Statement b does not give a clear idea of that the moon is a fascination to the people all over the
what is being emphasized. Statement c states a study world. Hence appropriate choice is (B). Choice (B)
made by WHO It is more appropriately a concluding
statement. It gives a conclusion to the topic. Hence it can 14. Statement 1 states about the serious threat to ecosystem.
precede statement 6. Statement a gives the reasons for the From the choices it is understood that b, d or a may follow
behavioural problems mentioned in 1. Hence statement a statement 1. If we look at statement d it says that
follows statement 1. Further it is followed by ‘b’ which
suggests the easiest to overcome those problems, radioactive elements cause damage to both environment
Statement d says that the social changes in society give and humans. It cannot follow 1 immediately because the
rise to many problems. It is a continuation of the ideas word ‘radioactive’ has no direct relation with statement 1.
expressed in a and b which precede it. Further c is the Hence choice (B) can be ruled out. If we look at statement
concluding statement. Choice (A) A the words “These sources….” do not give a clear idea of
the sources. Hence choice (D) can be ruled out. Between
10. The key word in statement 1 is ‘drought’. If we look at the
choices (A) and (C) we have to decide between ba and bd.
choices either a, b or d may follow statement 1. If we look
at statement a the words “This is …..” do not give clear b states about the sources of energy. a states about the
expression of the idea expressed in 1. Hence a cannot harmful effects of these sources. Thus ba is a better
follow statement 1. Thus choice (B) can be ruled out. If we combination. It is found in choice (A). The idea expressed
look at statement d it states the effects of drought even in b is a continuation of the idea expressed in 1 and further
before defining it. Thus choice (C) can also be ruled out. a explains the idea expressed in b. This combination is
Now from choices (A) and (B) we understand that b follows
found only in choice (A). Choice (A)
statement 1. b defines ‘drought’ and describes the situation.
Thus it explains the idea expressed in 1. Now we have to
decide between a and d to follow b. bd is the most 15. Statement 1 says that youngsters seem to get excited to
appropriate combination because the words “to sell their work. If we look at the statements, a is the most appropriate
only means of survival……” in d refer to the words ‘crippling statement to follow 1. The words “The thrill of working” in a
and dehumanising” in b. The combination bd is found in are a continuation of the idea expressed in 1. Further the
choice (A). Choice (A) words “This is a syndrome ….” in c explains the idea
expressed in a. Further d says why many of the students
11. Statement 1 is about “global warming” which is the key go for a part time job. Thus acd are logically related
word. If we look at the statements, b is the most statements. This option is found only in choice (D).
appropriate statement to follow 1. The word “global Choice (D)

Triumphant Institute of Management Education Pvt. Ltd. (T.I.M.E.) HO: 95B, 2nd Floor, Siddamsetty Complex, Secunderabad – 500 003.
Tel : 040–40088400 Fax : 040–27847334 email : info@time4education.com website : www.time4education.com SM1002114/83
Explanatory notes for questions 16 to 30: 21. Statement A says migration of birds has been an
astonishing phenomenon. Statement 3 is the most
16. Statement A says the earliest reasons to travel is to appropriate to follow A because the words "where to go"
develop trade and commerce. The words “Another strong refer to migration mentioned in A. Further if we look at
impulse …” in 2 state that it is a continuation of the idea statement 2 it says how do birds go to various places which
is an answer to statement 3. 4 says birds have extra-
expressed in A. A says in the olden days people travelled
ordinary powers to know their geographic position and this
to develop trade and commerce. 2 says another reason is idea finds a continuation in E. Hence 4 should precede E
to take a pilgrimage to holy places. Thus 2 follows A. Hence 3214 is the most appropriate answer. Ans: (3214)
Further, in 4 the words “such a visit” emphasizes the idea
expressed in 2. Thus 2 and 4 go together. Further 3 says 22. A states about the cooling powers of water and statement 3
gives an example of the idea expressed in A. It is a striking
that our epics emphasize on people’s longing to visit
example because where there are deserts the cooling
temples. This is an extension of the idea expressed in 4. powers of water make the area around the river the centre
Hence 2431 is the logical order. Ans: (2431) of life. Now we have to decide between 3 and 4 and 3 and
1. If 3 is immediately followed by 4 we do not find any
17. Statement A says history is a collection of myths and coherence between the statements. But statement 1 says
magics. If we look at statement 1 “Some of them” refer to the calendars of Egyptians were based on the flooding of
collection of myths and magics stated in A. Hence 1 follows the Nile. Hence 3 and 1 is the better option. 4 contains
A. We have to decide between 1 followed by 3 or 1 what is stated in 1. 2 is the appropriate statement to
followed by 4. Statement 4 gives an example of the idea precede A. Hence 3142. Ans: (3142)
expressed in 1. Hence 1 followed by 4 is a better option. If
we look at statement 3 it does not logically follow 1. 1 and 4 23. Statement A says Indian culture is unique. Statement 3
go together. Hence 1432 is the logical order of sentences A says that in no other part of the world such cultural plurality
and E. Ans: (1432) co-existed other than in India. Hence 3 is the appropriate
statement to follow A. Either 1 or 4 can follow statement 3,
18. Statement A states about the percentage growth in Indian but 1 followed by 2 is a better choice than 4 followed by 2.
economy. Statement 1 says “India did it ….” Which does 1 says shias and Sunnis do not kill each other just as their
not explain the idea mentioned in A. Hence 1 cannot follow counterparts in our neighbourhood. 2 emphasizes on the
A. In statement 2 the words “….You can do it” do not give synergy of Hinduism and Islam which is a continuation of
us a clear idea of what is being emphasized in the the idea impressed in 1. If we take the combination 2 and 4,
statement. Hence 2 cannot follow statement A. Statement 3 4 says Indian Christianity remained free from confessional
says “India has to do more …” which is a comparative conflicts. 2 refers to the synergy of Hinduism and Islam.
statement i.e. some other statement should have preceded Thus statements 4 and 2 are not logically related. Hence
it. Statement 4 says this resurgence “which refers to the 1 followed by b is 2 better choice 3412 is the appropriate
percentage growth in Indian economy as mentioned in A. sequence Ans: (3412)
Thus 4 is the most appropriate statement to follow A.
Further, in 2 it is mentioned how such a resurgence was 24. Statement A says time is the factor which governs our lives
possible. Followed by statement 1 which says that India to a great extent. 2 is an emphasis of what is stated in A. If
has proved it. Statement 1 is a continuation of the idea we look at the statements, 1 and 4 go together. 4 says time
expressed in A. Further it is followed by 3. Hence 4213 is is a matter of great importance. 1 says the almanac is
the appropriate sequence. Ans: (4213) central to the planning of most events and festivals. Thus,
the almanac states the importance given to time. Further
19. Statement A says genetically modified products are viewed the idea expressed in 1 finds a continuation in 3. The word
with great hostility. If we look at the choices, statement 1 ‘it’ in 3 refers to ‘the almanac’ stated in 1. Statement 3 finds
cannot follow A because it says GM products have a continuation in E. The word ‘it’ in E is a continuation of
undergone intensive field trials. It does not follow the idea the idea expressed in 3. Hence 3 precedes E. Thus, the
expressed in A. 2 cannot follow A because the words “tried correct sequence 2413. Ans: (2413)
out here” do not give a perfect idea of what is being said.
3 is more appropriately a concluding statement in which the 25. 2 is the most appropriate statement to follow statement A
condition of India is compared with that of China. Hence it because in 2 it is said that agricultural production has
cannot follow A. 4 is the most appropriate statement to become static after the green revolution, which is the
follow A. 4 says in a poor country like ours, technology reason for the desperation for a good monsoon. 2 followed
could make a difference between life and death. It is a by 4 is a better choice. These statements convey that in
continuation of the idea mentioned in A. The words spite of the fact that agricultural productivity has been
“conspiracy to undermine us” find a continuation in static, 25 percent of India’s GDP (Gross Domestic Product)
comes from farm sectors. If we look at statement 3 we find
statement 4. 2 carries forward what is stated in 4. ‘Here’ in
2 links with ‘country’ in 4. 1 which begins with ‘but’ a reference made to the people below the poverty line and
contrasts with what is stated in 2. 3 elaborates on what is in E it is said that they are the ones whom politicians count
stated in 1. Hence 4213 is the correct sequence. during elections. Thus 3 is the most appropriate statement
Ans: (4213) to precede E. The logical sequence 2413. Ans: (2413)
26. 3 is the better choice to follow A because in A it is stated that
20. Statement A says wildlife has been hunted for a very long plants and trees do not just stand doing nothing, rather (as
period when forests covered vast areas of land. If we look given in 3) they absorb the soil moisture through their roots
at the choices, the statements 1, 2 or 4 may follow which is unseen to the naked eyes.3 followed by 1 is a better
statement A. 1 cannot follow statement A because the choice because, the process stated in 3 is named in 1. If we
words “myth, legend and religious beliefs” do not find any look at statement 2 we find a reference to “convectional
reference in statement A. Statement 2 says hunting was for rainfall” which is again found in E. Hence 2 is a better
commercial interest. It cannot directly follow statement A. It statement to precede E. 3142 is the logical sequence of
does not emphasize the idea expressed in A. Statement 4 sentences to come between A and E . Ans: (3142)
says social taboos and religious sentiments and hunting 27. 3 is the better choice to follow A because in A it is stated
ethics also controlled the wildlife. This is 1 continuation of that the government of India urged the Royal Bhutan
the idea expressed in A. Hence 4 follows A. 4 and 2 is a Government to clamp down the militant camps” and in 3 the
better combination because the words “hunting ethics” find words “the issue” refer to the strict action to be taken
a continuation in 2 which says hunting was for living and against the militant camps. 4 elaborates on. Further
not for commercial purpose. Statement 3 is an appropriate statement 3 should be followed by 1 and 2 is a continuation
statement to precede E. Ans: (4213) of 4. Ans: (3142)
Triumphant Institute of Management Education Pvt. Ltd. (T.I.M.E.) HO: 95B, 2nd Floor, Siddamsetty Complex, Secunderabad – 500 003.
Tel : 040–40088400 Fax : 040–27847334 email : info@time4education.com website : www.time4education.com SM1002114/84
28. 2 is the appropriate statement to follow A because in A it is Statement e: This follows a stating that there are different
stated that the topic “Reservation” evokes strong feelings types of modes of communication
and in 2 the words “so acrimonious” tell us about the Statement a: The various modes of communication are
intensity of the strong feelings. Further, 4 is the better explained in a
choice to follow 2 because in 3 it is stated that the debate Statement c: Whether it is bodily expressions or speech
on “the policy” generates animosity and in 4 it is stated that that comes first in contributing to the communication
it is a proper debate on the policy. Further, it is followed by system as discussed in c.
1 and 3. Ans: (2413) Statement b: This concludes the passage focussing the
significance of gestures. Choice (D)
29. 4 is the most appropriate choice to follow statement A,
because it begins with a reference to the civilized world. In 4. Statement b: This statement begins the para by opening
4, the reference is to the caveman i.e. a comparison is the topic individual perception’.
made between the lifestyle of civilized people and the Statement e: ‘This is what makes perception’ in e can refer
uncivilized. 1 cannot follow A because the pronoun ‘he’ to no other statement in the passage than b. Statement c :
does not give us a clarity of the topic. For the same reason This statement carries the paragraph forward.
2 also cannot follow A. best choice is 4. Now we have to In Statement a: Tis statement is a continuation of c.
4 it is stated that “the cave man lived by instincts”, which if Statement d: We cannot take ‘d’ excepting as a concluding
followed by 3 seems to be incoherent, because in 3 it is sentence. Choice (B)
stated how his reflexes work without giving an introduction
to the tem reflexes. Hence 4 followed by 1 is a better 5. Statement d: It is evident that d only proposes the general
choice. Ans: (4123) topic – ‘gender roles, rules and expectations’.
Statement b: This statement is a corroboration of d.
30. 1 is the appropriate choice to follow statement A because in Statement e: This carries forward what is staed in b.
A it is said that in the present society, education and Statement a: This is a continuation of e.
degrees have acquired more symbolic meaning that they Statement c: “Different hats” is a continuation of the “many
deserve. In 1 the words “status conscious society” reflect faces and abilities” mentioned in a.
the meaning expressed in A. Further statement 1 is Hence, dbeac. Choice (C)
followed by 4 in which the words “rank and reputation” give
6. Statement e: This is the opening statement. ‘The
meaning to the phrase “status conscious society” used in 1.
quintessence of life is ‘stated in a single word in the
3 and 2 go together in which it is stated that the competitive
statement ‘e’.
spirit exists not only among students but also among
Statement b: The statement explains how that ‘harmony’,
parents. Thus 2 is a better statement to precede E.
the essence of life is thoroughly disturbed.
Ans: (1432)
Statement a: This statement follows b stating that the
Exercise – 13 pressures of modern existence are faced by the younger
generation as well.
Explanatory notes for questions 1 to 15: Statement d: This statement concludes the passage on
a sad note that even the carefree life associated with the
1. The focus of the passage is ‘diamonds’ which have been youth is just a thing of past only. Choice (A)
immortalised in Indian mythology’. The words ‘whether or
7. Statement c: This statement proposes the discussion on
not it’ in b, ‘so brilliant’ in c, ‘tales like this’ in d clearly tell us
‘Nepal’s breath taking beauty’ along with its ‘corrupt
that they are not independent sentences and have
politicians’. The rest of the sentences explain how the
a reference.
people of Nepal are suffering from a ‘farcical democracy’.
Statement a: This statement begins the paragraph
Statement e: This statement follows c explaining how Nepal
explaining how diamonds have a brilliance that fires the
is cursed.
imagination of the soul.
Statement a: This is a substantiation of what is stated in
Statement c: This statement is a continuation of a as the
c and e.
word ‘brilliant’ can be linked to ‘brilliance’ in a.
Statement d: This statement follows a explaining the
Statement e: This statement corroborates c explaining how
reason why the money does not reach the grassroots.
diamonds came to be associated with the mystical and
Statement b 1: This statement concludes the paragraph.
magical
Choice (B)
Statement d: This statement follows e, linked by the word
‘tales’. 8. Statement c: This statement , which speaks which speaks
Statement b: This statement is conclusive in nature. Hence of the importance of the financial markets which are
the sequence is acedb as in choice (B). Choice (B) occupying the centre stage, is the opening statement.
Statement a: The opening words of a ‘the reason is obvious –
2. Statements d cannot start the para because it begins with these markets’ can refer to no other statement than c
“but” e cannot start the para, because the word “the prey” Statement e: This statement carries the paragraph forward
indicates that the word has already been mentioned in a stating that a career in finance is no longer perceived as
previous statement. a record keeping job.
Statement b: This statement begins the para giving an Statement d: This is a continuation of e.
introduction to what follows in the subsequent statements. Statement b: This statement concludes the discussion by
Statement e: This statement follows by bringing out the focusing on the significance of a career in finance.
similarities in the behavioural patterns of human and Choice (B)
animals.
Statement d: This statement carries the idea forward by 9. The opening and closing pronouns ‘it’ and ‘that’ in a and c
presenting a contrast. respectively reject the sentences to be the openers. The
Statement a : ‘This factor’ refers to the factor- that predator conjunctions ‘and’ and ‘but’ in d and e must be nothing but
and prey are of the same species in the case of human the continuation of the previous sentences.
beings. Statement b: This is the obvious statement that can begin
Statement c: This statement, which begins with ‘thus’ is the passage.
conclusive in tone. Statement a: The pronoun ‘it’ in a clearly refers to
Hence, bedac is the corrects sequence. Choice (A) ‘an advertisement’ of b.
Statement c: This statement carries the idea forward.
3. Statement d: Though the statements bcde stand a chance Statement e: This is a logical continuation of c.
for the opening sentence at first glance, it can be clearly d Statement d: This statement follows e linked by the word
only that is suitable as it introduces the topic ‘message’.
‘communication’. Hence, baced is the correct sequence. Choice (C)
Triumphant Institute of Management Education Pvt. Ltd. (T.I.M.E.) HO: 95B, 2nd Floor, Siddamsetty Complex, Secunderabad – 500 003.
Tel : 040–40088400 Fax : 040–27847334 email : info@time4education.com website : www.time4education.com SM1002114/85
10. The pronoun ‘they’ in the statements b and e and the Explanatory notes for questions 16 to 30:
adjective the latter in c do indicate us that they are in
continuation of a previous statement. 16. Statement 3: This statement alone proposes the topic
Statement d: This statement, which proposes a historical ‘headache’ and the rest of the sentences explain the ‘types
fact related to the implements being used by the human of headache’ and the percentage of people suffering from
beings during the course of evolution, is the opening it.
statement. Statement 1: This supports the idea expressed in 3.
Statement a: This is just a statement of fact mentioned in Statement 4: This statement corroborates what is stated in
relation to d. 3 and 1.
Statement c: ‘The latter, here, refers to the ‘handy Statement 2: This is a continuation of 4.
sharpened stone’ in a. Statement 5: This statement closes the passage predicting
Statement e: This statement is a continuation of c. the grave impending danger ahead for mankind as far as
Statement b: This statement concludes the para explaining this problem – ‘headache’ is concerned. Ans: (31425)
how the pictures evolved into systematized symbols.
17. Statement 4: This statement introduces the topic ‘writing
Hence statements daceb form a sequence. Choice (A)
instrument’ and the role of the quill pen that was introduced
700 years ago only.
11. The opening words ‘but’, ‘for this’, we must use this’, ‘these
Statement 1: This statement follows 4 by describing the quills
are’ in the statements a, b, c, and d respectively declare
Statement 2: This is a continuation of 1.
that these sentences are just the explanations of the
Statement 5: This is a continuation of 2.
previous sentences.
Statement 3: This statement follows 5. .
Statement e: This is clearly the opening statement.
Hence 41253. Ans: (41253)
Statement c: ‘This drought’ refers to ‘the drought’
mentioned in e. 18. The words – "while" and "this" which open the sentences 2
Statement b: This is a logical continuation of c. and 5 convey that they are in continuation of the previous
Statement a: This is a continuation of b. statements. The word ‘this migration’ in 1 doesn’t refer to
Statement d: This statement concludes the paragraph stating the preceding statement 3. Hence 3 cannot be the opening
that the droughts are ‘government-made disasters’. sentence though it may appear so.
Choice (B) Statement 4: This is clearly the opening statement.
Statement 1: This statement follows 4 by supporting the
12. The opening conjunction ‘yet’ and the pronoun ‘this’ in a idea of ‘unprecedented migration in 4 with the ‘rise in the
and b respectively and the words ‘complaints against the number of slums’.
above’ in e state that they refer to some statement and Statements 5 and 3 continue what is stated in 4 and 1.
hence they are not independent. Statement 2: This statement is conclusive in nature.
Statement c: This statement begins the para by mentioning Hence 41532 Ans: (41532)
an observation.
Statement d: The example that is quoted in d refers to the 19. ‘But’ and ‘all this’ used as the opening words in the
statement c. statements 3 and 5 tell us that they are dependent
Statement a: This statement follows d mentioning that the statements. Though 1 also seems to be the beginning
practice of dowry has flourished despite the amendment of sentence of the passage, the words – ‘in other countries’ in
the Anti Dowry Act. 4 fail to refer to 1 as they refer to 2 alone which proposes
Statement e: This is a continuation of a. discussion on ‘health come delivery system in India’ in
Statement b: This statement concludes the paragraph relation to other countries.
stating that the scenario holds good for every punitive law Statement 2: This conveys the main idea o the passage.
in the country. Choice (A) Statement 1: This follows 2 by stating how the health care
delivery system in other countries is different from that in
13. Statement d :The beginning words ‘they’, ‘so’ ‘they’, ‘there’ India.
in the statements abc and e respectively proclaim that they Statement 3: This carries followed what is stated in 4.
are not independent sentences. Hence the clear choice is – Statement 1: This is a continuation of 3.
d. Statement 5: This is the concluding statement
Statement c: ‘They’ refers to the corrupt in d. Ans: (24315)
Statement e: This mentions why they should protect each
other. Hence, it follows c. 20. Statement 4: Only 4 is the appropriate choice to begin the
Statement a: This is a continuation of e. paragraph as it gives an introduction to the subsequent
Statement b: This is conclusive in tone. sentences.
Hence, the sequence is dceab. Choice (A) Statement 3: This follows 4 by stating that there are
advantages and disadvantages of conducting telephonic
14. Statement c: This statement begins the paragraph by putting interviews.
Statements 1 and 5, in that order, follow 3 by elaborating on
forward an observation.
the advantages and disadvantages of telephonic interviews.
Statement e: ‘They’, here, refers to the Indians in c. Statement 2: This is the concluding statement of the
Statement a: This follows e by presenting a contrast. paragraph. Hence 43152 is the correct sequence.
Statement d: ‘This’ , here refers to the changed scenario Ans: (43152)
mentioned in a.
Statement b: This statement concludes the paragraph by 21. Either 1 or 4 should begin the paragraph. Statement 1
suggesting a solution. Choice (C) cannot open the paragraph because the words “the project”
do not give a clear idea of what is being said.
Statement 4: This is ideally the opening sentence. Now we
15. Statement ‘d’: This statement highlights the ‘era of genetic
have to check for the appropriate statement to follow 4. 4
nutrition’ in relation to the publication of a ‘human genome’. says the idea for linking the major rivers is to form a network.
Statement b: This carries the paragraph forward. Statement 2: 2 is the most appropriate statement to follow
Statement e: This statement follows b linked by the word 4. The words “that should not only solve the problem” in 2
genetic makeup. refer to the idea expressed in 4. Hence 4 and 2 is the better
Statement c: This is a substantiation of what is stated in e. combination.
Statement a: This statement concludes the paragraph Statement 3: This statement follows 2 by mentioning the
impediment to execute the project i.e. the lack of funds.
talking about nutrigenomics which looks at the effect of
Statement 1: This is a continuation of 3.
nutrition at the molecular and genetic level. Statement 5: This statement is conclusive in tone.
Choice (D) Ans: (42315)
Triumphant Institute of Management Education Pvt. Ltd. (T.I.M.E.) HO: 95B, 2nd Floor, Siddamsetty Complex, Secunderabad – 500 003.
Tel : 040–40088400 Fax : 040–27847334 email : info@time4education.com website : www.time4education.com SM1002114/86
22. Statement 2: This is the most appropriate statement to Statement 2: ‘This’ in 2 refers to the idea expressed in 5.
begin the paragraph. Statement 4: This is the appropriate conclusion for the
Statement 5: Those Satanic Mills’ refers to the cotton mills paragraph. Hence 31524 is the correct order of sentences.
mentioned in 2. Ans: (31524)
Statement 4: ‘His’ , here, 4 refers to ‘Gandhi’ in 5. Hence 4
follows 5. 29. Statement 1 cannot start the paragraph because the words
Statement 1: This is an extension of what is stated in 4. “the white Elephant has a very exalted position” tell that the
Statement 3: This statement is conclusive in nature. Hence word “the white elephant” has already been introduced
25413 is the logical sequence. Ans: (25413) earlier. Statement 3 cannot open the paragraph because
the words “Among them .........” do not give a clarity of the
23. Statement 3: This statement, which gives an introduction to idea expressed.
what follows can open the paragraph. 3 says in Gujarat every Statement 2: This is the most appropriate statement to
house is said to be a shop or workshop. open the paragraph.
Statement 5: The word “Its” refers to Gujarat stated in 3. Statement 1: In 2, the key word is the “lore of white
Hence 3 and 5 is a better combination. elephant” further followed by 1 which has the reference of
Statement 1: ‘They’ in 1 refers to the people in 5. “The white elephant”.
Statement 4: This statement logically follows 1. Statement 4: This explains what is stated in 1.
Statement 2: This statement is a continuation of 1. Statement 3: “Them”, here, refers to the exotic beings in 4.
Hence 35142. Ans: (35142) Statement 5: This statement concludes the para stating
that the white elephant has always been considered pious.
24. In statement 1 the word “this” does not give a clear idea of
Hence 21435. Ans: (21435)
the topic. Hence it cannot open the paragraph. Statement 5
cannot open the paragraph because the word ‘he’ does not 30. Statement 3: This statement, which states that the story of
give a clear idea of whom the topic is all about. Krishna attracts the attention of most people, is the opening
Statement 2: This is clearly the opening sentence. We have sentence of the paragraph.
to decide between 4 and 5 to follow 2. Statement 1: This statement, which talks about the childhood
Statement 5: 2 says John Montagu did not like to be adventures of Lord Krishna, is a continuation of 3.
disturbed while he was at the gambling table. 5 is a Statement 4: This follows 1 by mentioning about one way of
continuation of 2. interpreting Lord Krishna’s adventures.
Statement 3: This statement follows 5 by explaining how Statement 5: This statement carries forward the paragraph
he solved ‘the problem’. by mentioning about another way of interpreting the
Statement 1: ‘This mundane arrangement refers to the adventures.
creation of the sandwich. Statement 2: This statement concludes the paragraph by
Statement 4: This statement winds up the paragraph. mentioning what the adventures signify at a deeper level.
Hence 25314 is the sequence. Ans: (25314) The logical sequence, therefore, is 31452. Ans: (31452)
25. Statement 4: This statement introduces the name Exercise – 14
Bankadagah and tells us about its location.
Statement 1: This follows 4 by further elaborating on Explanatory notes for questions 1 to 15:
Bankadagah.
Statement 3: This statement contrasts with 1 by stating that 1. Statement a: This statement begins the para by stating
the place which was once the pulsating capital of the what the death of a language means.
Sailodbhavas is now desolate and ruined. Statement d: This supports a by speaking about the
Statement 5: This statement carries followed what is stated importance of language by quoting Ludwig Wittgenstein
in the preceding statements. who said that the limits of one’s language were the limit of
Statement 2: This statement is conclusive in nature. Hence one’s world.
41352 is the appropriate sequence. Ans: (41352) Statement b: This statement corroborates what is stated in
a and d by citing the example of the death of Bo – one of
26. Statement 1 cannot begin the paragraph because it is a the ten languages spoken by the Great Andaman tribes.
continuation of some other statement. Statement 3 begins Statement e: This statement, which carries forward what is
with a connective hence it cannot open the paragraph. stated in b is a continuation of b.
Statement 2: Only 2 has to open the paragraph. Statement c: This statement is conclusive in nature.
Statement 4: This statement follows 2 talking about dbec is the correct sequence. Choice (B)
excessive patriotism. 2. Statement a states that the big powers are contemptuous
Statement 3: This says that if patriotism becomes blind it about the global ban on cluster bombs.
leads to fanaticism and sectional clashes i.e. the idea Statement d: This statement carries forward the idea
expressed in 4 finds a continuation in 3. Hence 4 and 3 is expressed in a by speaking about the arms manufacturing
the appropriate combination. countries’ refusal to accede to the pact, thus isolating
Statements 5 and 1 carry forward what is stated in the themselves from promoting international peace and
preceding statements. Ans: (24351) multilateralism.
Statement b: This statement, which speaks about the
27. Statement 5: This is the most appropriate statement to start extent of damage cluster bombs can cause and how cluster
the paragraph. because it says that the scientists must be bombs are capable of killing thousands of innocent
allowed to experiment on animals. civilians, is a continuation of d.
Statement 1: This statement follows 5 experimenting on Statement e: This statement talks about the success
laboratory animals forms an appropriate linking statement. recorded in the abolition of landmines which have a
Statement 1 says cure for any ailment has been discovered similarity with cluster bombs and the countries which used
after experimenting on laboratory animals. these landmines. Hence e follows d.
Statements 2 and 3 give examples of the ideas expressed in. Statement c: This statement concludes the para by saying
Statement 4: This statement is conclusive in tone. that the apology offered by the nations for the violations of
Hence 51234. Ans: (51234) the terms of engagement is very perfunctory.
Hence dbec is the correct sequence. Choice (A)
28. Statement 3: This statement begins the paragraph by
giving an introduction to what follows in the subsequent 3. Statement c: This statement is a continuation of a. a states
sentences. that many emerging economics are having inflows at such
Statement 1: This further elaborates on what is stated in 3. a high level as to pose a challenge to their macroeconomic
Statement 5: This statement follows 3 by carrying the idea management. c supports a by stating how the inflows have
forward. been possible.
Triumphant Institute of Management Education Pvt. Ltd. (T.I.M.E.) HO: 95B, 2nd Floor, Siddamsetty Complex, Secunderabad – 500 003.
Tel : 040–40088400 Fax : 040–27847334 email : info@time4education.com website : www.time4education.com SM1002114/87
Statement e: This statement carries forward the idea and 9. Statements c and e which carry forward the idea expressed
is , hence, a continuation of c. in a are a continuation of a.
Statement b: This statement is a continuation of e. Statement b: This statement, which speaks about the
Statement d: This statement corroborates what is stated in person who was responsible for Sauri’s prosperity and the
e and b by citing the example of India. initiative taken by him in lifting the place out of poverty, is a
Hence cebd. Choice (C) continuation of e.
Statement d: This statement concludes the para by saying
4. Statement a speaks about the scientists’ response to the that Jeffrey Sachs’ initiative seems to be working in Sauri.
media criticism and how they have adopted a defensive Hence cebd is the correct order. Choice (D)
attitude so far. Statement e : This statement follows a by
stating that the scientists are no longer able to defend 10. Statement e: ‘This potent combination of the old and the
themselves. The volume of criticism and the depth of doubt new’ in option e refers to the change which is being
refer to the criticism levelled against the scientists, stated in a. witnessed, at present, in India and the rich history, culture
Statement c: This statement, which says that the scientists and traditions mentioned in option a. Hence e follows a.
are now beginning to admit mistakes, is a continuation of e. Statement c: This statement contrasts with what is stated
Statement b: This statement ,which mentions what in e by using the word ‘however’.
prompted the scientists to reshape the way they conduct Statement d: This statement elaborates on the inequality
their work is a continuation of c. mentioned in c by referring to the unfair treatment meted
Statement d: This statement is conclusive in nature. out to women.
Hence ecbd. Choice (D) Statement b: This statement is a continuation of d. Hence
ecdb is the correct sequence. Choice (A)
5. Statement a states that mobile phones have brought voice
and data access to countless people.
11. Statement e: This statement is a continuation of a as ‘the
Statement d: This complements a by providing figures to
poignant moment’ in e refers to the scene described in a.
support what is stated in a.
The scene described in a is a scene from the film Gandhi
Statement b: This statement , which states how the mobile
by Richard Attenborough.
phones have been beneficial is a continuation of d.
Statement c: This statement is a continuation of e.
Statement e: This statement follows b by saying that
Statement b: This statement follows c by bringing about
because the mobile phones have been beneficial India
a contrast between leaders of the past like Mahatma
should pay more attention to this sector.
Gandhi and the present day politicians.
Statement c: This statement concludes the paragraph by
Statement d: This statement, which is the concluding
citing examples as to how mobile phones can open up new
statement, supports b by saying that the present generation
possibilities.
of politicians do not have the aim of serving people but they
Hence dbec is the correct sequence of sentences.
are more interested in wearing designer labels and
Choice (B)
zooming across cities in their cavalcades.
6. Statement a states that the second goal of economic Hence ecbd is the correct sequence. Choice (D)
planning i.e. the equitable sharing of the fruits of
development has remained elusive. 12. Statement a states the topic, i.e. moonlighting, on which the
Statement d: This statement supports a by saying that paragraph is based.
three quarters of the population are mired in poverty. Statement c: This statement follows a by mentioning how
Statement c: This statement follows d by saying that the moonlighting has become popular among the youth and
world financial crisis offers an opportunity to make growth how moonlighting helps them.
more inclusive. Statement b: This statement, which speaks about the
Statement e: This statement cites the example of the circumstances which provided an additional impetus to
Keynesian model in order to explain how it helped in moonlighting, follows c.
fighting the world financial crisis. Statement e: This statement follows b mentioning how
Statement b: This statement concludes the para by saying young employees are considering secondary jobs in order
that the Indian government has already initiated action in to be prepared for adverse situations like downsizing etc.
the right direction but more needs to be done. Hence dceb Statement d: This statement , which talks about the
is the correct order. Choice (A) negative aspect of moonlighting contrasts with the
preceding paragraphs.
7. Statement a talks about the Vikings’ penchant for building Hence cbed is the correct sequence.
new ships that helped them conquer new lands. Choice (B)
Statement c: This statement , which says that the Royal
Carribean International has Viking blood flowing through its 13. It is stated in statement a that new solutions have been
veins is a continuation of a. suggested to improve health care in rural India.
Statement e: This statement which elaborates on the Royal Statement e: This statement follows a by mentioning one of
Carribean International is a continuation of c. the solutions suggested.
Statement b: This is a continuation of is a continuation of e. Statement c: This statement follows e by mentioning some
Statement d: This statement concludes the para by more solutions suggested. The words ‘also suggested’ in c
describing the Oasis of the Seas, the largest cruise ship indicate that c is a continuation of e.
built by the Royal Caribbean. Choice (D) Statement b: This statement, which says that ‘most of these
8. Statement a says that livestock rearing is a key livelihood recommendations are not new’ has to follow c.
and risk mitigation strategy for small farmers. Statement d: This statement concludes the para by
Statement c: This statement supports a by mentioning the speaking about the cynics’ opinion. Hence ecbd.
statistics to prove that livestock is all set to be the engine Choice (D)
for growth for Indian agriculture. Statement e : This
statement is a continuation of c. 14. Statement a says that India has not seen any success at
Statement b: This statement follows e by saying that the highest level in its national game in three decades.
although livestock is seen as a provider of essential food Statement d: This statement follows a by saying that the
products, employment, household income etc., livestock euphoria about the recent win was only short-lived.
wealth is more equitably distributed when compared to Statement b: This statement, which explains why the
wealth, associated with land. euphoria was short-lived follows d.
Statement d: This statement , which says that livestock Statement e: This statement is a continuation of b.
rearing must be at the centre of stage in poverty alleviation Statement c: This statement is conclusive in nature as it
programmes, forms an ideal conclusion for the paragraph. analyses the reason behind India’s failure. Therefore, dbec
Hence cebd is the correct sequence. Choice (B) is the correct order of sentences. Choice (A)

Triumphant Institute of Management Education Pvt. Ltd. (T.I.M.E.) HO: 95B, 2nd Floor, Siddamsetty Complex, Secunderabad – 500 003.
Tel : 040–40088400 Fax : 040–27847334 email : info@time4education.com website : www.time4education.com SM1002114/88
15. Statement d: This statement, which begins with ‘the 22. Statement 2: This statement is a continuation of a as it
reforms’ is obviously a continuation of a as the reforms explains how the question has been settled.
mentioned in d refer to the spate of reforms mentioned in a. Statement 4: This statement is a continuation of 2.
Statement b: This statement, which speaks about some Statement 1: ‘The discovery’ in 1 refers to the discovery
more reforms is a continuation of d. mentioned in 2 and 4.
Statement e: This statement , which says that education is Statement 3: This statement concludes the para by
now getting the recognition due to it, is a continuation of e. speaking about the most significant part of the discovery.
Statement c: This statement is conclusive in nature. dbec Therefore the correct sequence is 2413. Ans: (2413)
is the logical sequence. Choice (C)
23. Statement 3: This statement follows a by explaining why
Explanatory notes for questions 16 to 25: the IMF is in an upbeat mood. (because economic recovery
is faster than estimated).
16. Statement 2: This statement is an extension of what is Statement 1: This statement is a continuation of the idea
stated in a. ‘That’, here, refers to the fact, that there is expressed in 3.
always hope even amid the worst devastation, mentioned Statements 4 and 2 follow 1 by providing statistical details
in a. of the economic growth witnessed in recent years.
Statement 4: This statement goes on to explain what the Hence 3142. Ans: (3142)
narrator saw.
Statement 1: This is a continuation of 4. 24. Statement a states that the banks are now facing concerted
Statement 3: This statement is conclusive in nature. regulatory action.
Hence 2413. Ans: (2413) Statement 4: This statement, which says that the banks were
expected to be more circumspect, is a continuation of a.
17. Statement 4: This statement carries forward the idea Statement 1: This statement follows 4 stating that instead
expressed in a, hence 4 is a continuation of a. of being circumspect the banks are continuing with their
Statement 2: This statement is a logical continuation of imprudent investments.
Statement 3 carries forward the idea expressed in 1.
what is stated in 4.
Statement 2: This statement concludes the para.
Statement 1: This statement is a continuation of 2. Hence 4132. Ans: (4132)
Statement 3: This statement, which cites the reasons
which are responsible for the failure of safeguarding of 25. Statement 3: This statement, which states that there is
antiques, ideally concludes the paragraph. Hence 4213 is evidence to prove that the first vertebrates started walking
the correct order. Ans: (4213) 385 million years ago is a continuation of a.
Statement 2: This statement supports 3 by stating that land
18. Statement 2: ‘The encomiums’ mentioned in 2 refer to the vertebrates evolved from fish.
applause showered on Krishnadeva Raya by Domingos Statement 4: This statement is a continuation of 3.
Paes. Hence 2 is a continuation of a. Statement 1: This statement concludes the para by
Statement 4: This statement, which mentions the reasons mentioning the latest discovery which supports the fish-
for the encomiums being showered on the king follows 2. tetrapod transition.
Statement 3 : This statement is a continuation of 4. Hence 3241. Ans: (3241)
Statement 1: This statement, which explains how
Krishnadeva Raya could expand his kingdom within a Exercise – 15
relatively short period is a continuation of 3.
Hence 2431. Ans: (2431) Explanatory notes for questions 1 to 20:

19. Statement 2: This statement is a continuation of a as it 1. The main points are:


supports a, which says that millions of Yemenis are 1. Just as in science, economists also make assumptions.
starving, by stating that vital deliveries of food and 2. To study a change in one variable, the others
assistance is being cut due to an absence of funding. variables must be constant.
Statement 4: This statement corroborates what is stated in 3. In economics, the variables cannot be controlled.
2 by mentioning the statistics provided by the U.N.
4. This makes it difficult for the economists to pass on
Statement 3: ‘Its World Food Programme in option, 4 refers
to the World Food Programme initiated by the U.N. Hence the findings to the administrator for application.
3 is a continuation of 4. Choice (A): This fails to mention the fact that if the change
Statement 1: This statement is conclusive in nature. in one variable are to be studied, the others must remain
Ans: (2431)
constant.
20. Statement a mentions the topic i.e. imposing a moratorium Choice (B): This is a distortion of the second point.
on executions by 2015. Choice (C): Correct. This option captures all the points of
Statement 3: This statement follows a by speaking about the passage.
the promising developments which add moral weight to the Choice (D): This option, which states that if the variables,
case for halting the executions forthwith. Statement 2 : This except one, are kept constant, is a distortion of the original.
statement supports 3 by citing an example. Choice (C)
Statement 1: This statement 1 is a continuation 2.
Statement 4: This statement concludes the para. 2. The key points are:
Hence the correct sequence is 3214. Ans: (3214) 1. Exercise burns calories and tone muscles.
2. Reduced in-take of food leads to slack skin.
21. Statement 2: ‘A similar stand’ in 2 refers to the decision 3. Walking, dancing and playing games are good
taken by India to release the Pakistani prisoners mentioned exercises.
in a. Hence 2 is an extension of a.
Statement 4: This statement follows 2 by stating that Choice (A): Correct. This option covers all the important
despite the government of India’s argument, the court stuck points.
to its decision. Choice (B): ‘Dieting and exercising together will cause
Statement 1: This statement follows 4 by giving a reason slack skin’ is a distortion of what is stated.
for what is stated in 4 – because reciprocity in this case is Choice (C): It is not stated in the passage that walking,
shameful. Hence 1 follows 4. playing etc. are not so good.
Statement 3: This statement is conclusive in nature. Choice (D): This option does not capture all the important
Hence 2413. Ans: (2413 ) points. Choice (A)
Triumphant Institute of Management Education Pvt. Ltd. (T.I.M.E.) HO: 95B, 2nd Floor, Siddamsetty Complex, Secunderabad – 500 003.
Tel : 040–40088400 Fax : 040–27847334 email : info@time4education.com website : www.time4education.com SM1002114/89
3. The main points found in the paragraph are: Choice (C): The words ‘the only achievement’ rule out this
1. Forests are of two kinds-temperate and tropical. option as it cannot be inferred from the passage that it was
2. Temperate forests are of two kinds deciduous hard wood the only achievement.00
forests and evergreen softwood coniferous forests. Choice (D): This option can be ruled out for the same
3. Europe was covered with forests once. reason given for C. Choice (B)
4. Forests have been cleared over those parts suitable
for agriculture and industry. 8. The main points are:
Choice (A): The last sentence of this option is a distortion 1. Biotechnology has played a key role in food production.
because it is stated in the passage only those parts of 2. High yielding strains of cereals engineered by artificial
forests which are found to be suitable for agriculture have selection, improved food situation.
been cleared.
3. Genetic engineering now means manipulating the
Choice (B): This fails to mention point 4 correctly.
Choice (C): This option, which fails to mention ‘agriculture’ genetic endowment of organisms.
is incorrect. Choice (A): Correct. This option captures all the important
Choice D: Correct. This is the best summary of the passage. points correctly.
Choice (D) Choice (B): This option fails to mention artificial selection.
Choice (C): This option fails to mention point 3.
4. The main points of the paragraph are: Choice (D): This option also fails to mention point 3.
1. In other nations there are many occupations of life – Choice (A)
politics, enjoyments of social life, wealth, power etc.
2. Religion is only one among the occupations. 9. The chief points are:
3. In India religion is the one and only occupation. 1. Consumer protection started in Great Britain to give
Choice (A): Correct. This captures all the important points. objective information on goods and services.
Choice (B): The last sentence in this option cannot be 2. The only information available was through advertisements.
inferred from the passage. 3. Organizations were set up to publish unbiased reports
Choice (C): This option, which states that it is ‘the essential on goods and services.
difference’ is incorrect because it cannot be understood Choice (A): This option, which is rather categorical in
from the passage that it is the essential difference. saying ‘wrong information’ is incorrect.
Choice (D): The last sentence of this option is a clear Choice (B): This option fails to mention point 2.
distortion of what is stated. Choice (A) Choice (C): This option is a distortion of point 1.
Choice (D): Correct. This option captures all the important
5. The main points of the passage are: points of the passage. Choice (D)
1. Knowledge of the earth’s interior is obtained through
lavas coming out of volcanoes. 10. The main points are:
2. The lavas consist of two types of materials-light acid 1. Criticism does not mean only fault finding.
materials called sial and heavy basic substances 2. A good critic points to merits and faults and criticism is
called sima. useful to both the artist and the public.
Choice (A): This option fails to mention point 2 completely. 3. He should also make a comparative analysis of this
Choice (B): Correct. This option is correct as it captures all work in relation to the previous ones.
the important points. Choice (A): This option is needlessly wordy.
Choice (C): This option which fails to mention the names of Choice (B): That ‘disgruntled critics find only faults’ is a
the two types of materials is incorrect. distortion because it is stated in the passage that they
Choice (D): This option is incorrect because it is not the occasionally find faults.
knowledge of the earth’s content but it is the knowledge of Choice (C): Correct. This is the appropriate summary of the
the earth’s interior that is obtained through the volcanic passage.
lava. Choice (B) Choice (D): The first sentence of this option is a distortion
of what is stated Choice (C)
6. The important points of the passage are:
1. The song of the cicada is produced by the male 11. The main points of the passage are:
2. The sound is produced by an organ with a lid which 1. Conservatism favours existing conditions and traditions.
has a vibrating membrane. 2. Edmund Burke developed the theory stressing the
3. The female has an ovipositor used to pierce the bark importance of tradition and history as the basis of
of the trees to lay eggs. progress and prosperity.
3. But he was not opposed to all changes and hence
Choice (A): This option is rather incomplete as it does not
conservatism of the 19th century was able to reform
capture the third point of the passage.
and progress.
Choice (B): This option fails to convey that the song
Choice (A): Correct. This option captures the summary of
producing quality among cicadas is unique to the males.
the passage appropriately.
Choice (C): Correct. This option captures all the important
Choice (B): The last sentence of this option is a distortion.
points.
Choice (C): This option is a distortion of point 3.
Choice (D): The last sentence of this option is rather
Choice (D): ‘He was not for changes’ is a distortion
ambiguous as it indicates that the membrane which
because it can be understood from the passage that he
controls the sound is found in females. Choice (C)
was not opposed to all change. Choice (A)
7. The main points are:
12. The chief points found in the passage are:
1. Fascism, a movement launched by Mussolini, is an
1. Microchemistry refers to methods used by
authoritarian doctrine.
manipulating small quantities of substances.
2. Fascists seized power and Mussolini established a
2. It is used in all fields of chemistry where the required
dictatorship in Italy.
information can be obtained using such small quantities.
3. Fascism sought to restore prestige and economic stability.
3. There are several causes responsible for stimulating
4. Development of the corporate state was the means of
its development – increasing demands from other
gaining control over economic life.
sciences, rapidly decreasing number of substances in
Choice (A): The last sentence of this option is a distortion large quantities and two world wars.
because it is not stated in the passage that economic 4. There are several advantages in using the technique.
stability was ensured. Choice (A): This option fails to mention point 3.
Choice (B): Correct. This option captures all the important Choice (B): Correct based on the explanation given at the
points correctly. beginning.

Triumphant Institute of Management Education Pvt. Ltd. (T.I.M.E.) HO: 95B, 2nd Floor, Siddamsetty Complex, Secunderabad – 500 003.
Tel : 040–40088400 Fax : 040–27847334 email : info@time4education.com website : www.time4education.com SM1002114/90
Choice (C): The last sentence of this option is a distortion 4. The great variability during periods of expansion
of what is stated. provides opportunities for evolutionary adjustment and
Choice (D): ‘The resulting technique has little advantage’, hereditary qualities may try out combinations, which
meaning it has no advantage, is a clear distortion of the may prove to be valuable.
passage. Choice (B) Choice (A): The second sentence in the passage is
a distortion of point 3.
13. The chief points are: Choice (B): Correct. This option covers all the important
1. Democracy is of, by, and for the people. points.
2. In ancient Greece it was direct with all people Choice (C): This option fails to mention point 4.
participating. Choice (D): This option does not cover all the points and is
3. It became representative democracy where people’s not the appropriate summary. Choice (B)
will was expressed through elected representatives.
4. Unanimity of views is rare in any society and hence 18. The main points found in the passage are:
democracy means majority rule. 1. A cheque must be dated.
5. But majority and minority both have some guaranteed 2. A bank will not pay a cheque before the date of issue
rights in a democracy. nor six months after it is issued.
6. Democracy must not deny the minority the right to 3. The cheque can be filled up by anyone but the
convert itself into a majority by peaceful persuasion. signature must correspond totally with the specimen
Choice (A): This option fails to mention point 6. held by the bank.
Choice (B): The last sentence of the passage is a distortion. 4. The sum in words and figures should be the same.
Choice (C): This option which stats that unanimity in any 5. Alterations should be signed.
community is impossible, is a distortion. 6. If a cheque is crossed it will be paid only to a banker
Choice (D): Correct. This is the precise summary of the and not across the counter.
given passage. Choice (D) Choice (A): Correct. This captures all the important points.
Choice (B): This is a distortion of point 2.
14. The main points of the passage are: Choice (C): The last sentence is a distortion.
1. Modern civilization is concerned with communication. Choice (D): ‘There should not be any alterations in the
2. For long distance communication short wave radio and cheque ‘ is a distortion of the words ‘ alterations must be
submarine cables are used. authenticated’. Choice (A)
3. The former can be disrupted by eruptions in the sum
and the latter is expensive and limited in reach. 19. The main points are:
4. The answer is satellites, which can receive and relay 1. Buddhism aims to inculcate unselfishness, though
signals. practice differs from precept.
Choice (A): Correct. This captures all the important points. 2. This has been responsible for the view that Buddhism
Choice (B): This option fails to mention point 3. is a system of ethics and Buddhism is an atheistic
Choice (C): The second sentence of this option is a distortion. view of life.
Choice (D): This option fails to mention 1. Choice (A) 3. The ethical teaching of Buddha has to be
considered in relation to enlightenment and release.
15. The important points are: 4. Buddhism involves restoration of human worth
1. Satyagraha means clinging to truth, it is the truth force whereas Buddha’s contemporaries were preoccupied
or soul-force. with otherworldly matters.
2. It is a way of life for one who holds to God and 5. The essence of Buddhism is the teaching about release
dedicates his life to God. through enlightenment that overcomes ignorance.
3. Truth can be attained by non-violence and service to all. Choice (A): This option is a distortion of point 4.
4. Evil is resisted through Satyagraha. Choice (B): The first sentence of this option is a distortion
5. To acquire this force, one must free him of all ill will of point 1.
towards everything including even insects. Choice (C): This option fails to mention point 4.
Choice (A): This option does not mention the first point. Choice (D): Correct. This option captures all the major
Choice (B): This option does not mention the second point. points. Choice (D)
Choice (C): Correct. This is the appropriate summary of the
20. The main points are:
passage.
1. Chess is a skill game of two players using a
Choice (D): The last sentence of this option is a distortion
chequered board and thirty-two special pieces.
of what is stated. Choice (C)
2. The origin of the game is said to be Persia or India.
16. The main points are: 3. The game was first played in the seventh century.
1. Modern industrial society is too large and complex for 4. The game is considered as a game of manoeuvre by
simple systems of ownership and direct control by all kings.
the people. 5. The game is linked with Buddhists who believed that
2. Two methods are practicable. slaying of men in a war was sinful and welcomed the
3. A centrally directed economic planning. game as a substitute for warfare.
4. Decentralized ownership of property, where there is a Choice (A): This option fails to mention point 4.
free market. Choice (B): This option is a distortion of point 4.
5. A third method – a market system in a state directed Choice (C): Correct . This option covers all the important
economy is a recent development. points.
Choice (A): This option fails to mention point 5. Choice (D): This option does not cover all the points.
Choice (B): This option, which is rather categorical is Choice (C)
saying that the third point is possible, is a distortion.
Choice (C): Correct as it captures all the points. Explanatory notes for questions 21 to 25:
Choice (D): Choice (C)
21. The following are the major points of the passage:
17. The main points found in the passage are: (1) The HIV infection is fast spreading in Europe so
1. The plant and animal populations are subjected to mobile clinics are pressed into service to battle it.
violent fluctuations in numbers. (2) The fact that two people, who tested negative, prove
2. These may be due to climatic cycles but are usually that the infection is declining in most of the world.
generated automatically. Choice 1: Correct. This option captures the summary of the
3. When environment is favourable the number of a passage most appropriately.
species increases, and variation survives when it is Choice 2: In this option, the facts are wrong. There's only
not, so there is contraction. one NGO mentioned and therefore this option is dismissed.
Triumphant Institute of Management Education Pvt. Ltd. (T.I.M.E.) HO: 95B, 2nd Floor, Siddamsetty Complex, Secunderabad – 500 003.
Tel : 040–40088400 Fax : 040–27847334 email : info@time4education.com website : www.time4education.com SM1002114/91
Choice 3: This option assumes that the two people who sentence has to be: This condition is commonly referred to
slipped into the side cabin are drug addicts, that is, junkies. as ‘heart failure.' Any other option will spoil the unity of the
The passage has no proof that they are junkies. So, option paragraph. Choice (A)
(3) is disqualified.
Choice 4: assumes that because two patients tested 2. The paragraph talks about the importance of cockroach.
negative, the HIV/AIDS infection is globally falling. That Through its very existence. It does not render service as
may not be true. Only option (1) convincingly captures the such. The absence of it will make diseases unmanageable.
essence of the passage. The solution is (1). Ans: (1) So, the concluding sentence should be: You understand
that its role in the world is more important than yours.
22. The following are the major points of the passage: Choice (A): It’s role in maintaining ecological balance, is not
(1) Getting advice on travel and finding one's way to a bus discussed in the passage.
stop or train station is one's own responsibility. Choice (B): Correct based on the explanation given above.
(2) Services such as Whim intend to facilitate seamless Choice (C): This runs contrary to what the passage is trying
travel everywhere for all. to convey.
Choice 1: This option has left out certain steps involved in Choice (D): This option is rather far-fetched and cannot be
the travel so this option is not entirely suitable as the answer. understood from the information given in the passage.
Choice 2: This option doesn't even mention at least one Choice (B)
hassle, so this option isn't appropriate.
Choice 3: Correct. This option captures most of the points 3. The sentence ‘It is Life’s change agent’ leads to the
and sounds comprehensive and conclusive. conclusion: It clears out the old to make way for the new.
Choice 4: This option is too general and hence cannot be Hence option D is the precise answer.
accepted. Ans: (3) Choice (A): This is not in sync with the penultimate
sentence of the para.
23. The key argument of the passage is that there were several Choice (B): This can be eliminated for the same reason
methods of getting rid of unwanted children in late-Victorian given for A.
England. Choice (C): This is inappropriate when compared to option D.
Choice 1: states that baby-killing was a method used in Choice (D): Correct based on the explanation given at the
England to get rid of unwanted children. The statement isn't beginning. Choice (D)
right because many babies died as a result of various
methods used by them. 4. Suggestions like, ‘challenging all stereotypes’, ‘shows off
Choice 2: Correct. This option portrays the facts appropriately. her wrinkles with élan’ and ‘which is far more graceful’ help
Choice 3: This option is rather imprecise as it does not to choose the option ‘She thinks that it is time the world lets
cover certain essential points. older women age gracefully.’ The other options are not
Choice 4: This option states that criminal methods were related to what is stated in the paragraph. Choice (C)
used to get rid of unwanted children. There's no mention of
the word "criminal" in the passage; this is a distortion of the 5. Reference to Graham Bell and the sentence beginning
original text. Ans: (2) ‘Moreover, he envisioned...’ prompt us to choose the option
‘As a restless diviner of the digital future, Jobs made things
24. The key argument of the passage is that Mr Peres used his for people before they knew they needed them.’
skills to help his countrymen by brokering arms deals and Choice (B)
circumventing arms embargoes. Besides he bargained
hard, shamed rich countries and cajoled rich sympathizers. 6. Comparing the experimental findings of quantum theory to
Choice 1: Correct. This option captures all the essential points. the discovery in the 15th century that the world was not flat
Choice 2: This option says that Mr Peres used his imprint leads us to choose the option: These findings have
and precociousness to broker arms deals and the rest. But increased our doubt and uncertainty about traditional
it fails to convey this was done – by compromising on rules. physical explanations of the universe’s genesis and
So this isn't fully conclusive. structure. The suggestion is that the findings, like the
Choice 3 : This option contradicts the passage because it discovery in the 15th century, can shake the very foundation
says Mr Peres transcended barriers and obstacles, which of knowledge. Only B matches with this.
the passage doesn't say. So, this option too isn't suitable. Choice (A): This is the converse of what is stated.
Choice 4: The second sentence in this option doesn't say Choice (B): Correct based on the explanation given at the
what he gained by bargaining hard and shaming rich beginning.
countries and at what expense. So option (4) is also Choice (C): ‘Obscured our doubt’ does not make sense in
eliminated. Ans: (1) this option.
Choice (D): This option also runs contrary to what is being
25. The key argument of the passage is that more pressure conveyed. Choice (B)
and pulls are at work hampering OPEC's unity.
Crude oil produced outside OPEC hampers its unity. 7. The writer uses words like ‘concern’, ‘fear’ and ‘hackles’
Choice 1: The first sentence of this option is a bit while discussing talking in a dyadic relationship. Then he
irrelevant and the passage doesn't say why there's disunity says this appears ludicrous considering the importance of
in OPEC. So this option is ruled out. communication in any relationship. So, the logical
Choice 2: The first sentence of this option isn't mentioned conclusion can only be: Yet, when it comes to serious
in the passage and it has cited only one reason for OPEC's talking, we all seem to go slightly on the defensive. Other
disunity. So, this option isn't fully appropriate. options are not related to the passage. Choice (A)
Choice 3: This option deviates a bit from the passage so it
8. The passage talks about the need for positive company.
is left out.
This is the only option that talks about positive company.
Choice 4: Correct. This option captures the lack of unity,
All other options are about other things. So, the only
rivalry between Iran and Saudi Arabia and the crude oil
possible option is: In order to inculcate positivity, it is
produced outside the cartel which affect the unity of OPEC.
imperative that your friends circle is a positive, energetic,
Ans: (4)
and a happy bunch. Choice (D)
Exercise – 16
9. The suggestion that Machu Picchu and Huyana Picchu
Explanatory notes for questions 1 to 25: form a backdrop to the Inca Citadel and that the ruins are
tucked away out of sight until one is actually upon them can
1. The opening sentence introduces the topic – heart failure. be the clues to choose the option, No wonder they were
Then the writer moves from known to unknown – from never found for so long.
‘pump’ to ‘blood providing nutrition to various organs of the Choice (A): Correct based on the explanation given at the
human body’, developing the concept. So, the concluding beginning.
Triumphant Institute of Management Education Pvt. Ltd. (T.I.M.E.) HO: 95B, 2nd Floor, Siddamsetty Complex, Secunderabad – 500 003.
Tel : 040–40088400 Fax : 040–27847334 email : info@time4education.com website : www.time4education.com SM1002114/92
Choice (B): This option is unrelated. 19. Though freedom of speech and freedom of information
Choice (C): This is also not pertinent in the given context. have their advantages, there are barriers to achieving these
Choice (D): This can form a part of another paragraph. ideals. However, as the balance of rural and urban
Choice (A) population changes (which is happening ,as the statement
indicates – “as people continue to migrate” –, deference to
10. The paragraph says that ‘talking’ is one-sided and the other authority would also change. Choice (B), which conveys
person is only a passive recipient. What he/she says is not this is appropriate.
taken in. So, the natural conclusion is: And finally, we Choice (A): This brings in a new idea.
erroneously believe that we have achieved the object of Choice (B): Correct based on the explanation given at the
good communication, agreement. The other options are beginning.
unrelated to the contents of the passage. Choice (C) Choice (C): ‘Foreign newspapers are not relevant here.
11. The given para ends with how Turkey had taken important Choice (D): This is not relevant for the logical completion of
steps to ensure their fit into the EU. The concluding the given paragraph. Choice (B)
sentence should speak about the same readiness and also 20. (A) is the only logical conclusion to the passage because if
about the people who will be speaking on behalf of Turkey. such news is being published in the newspapers in a
Option D is the best choice as it satisfies this condition. country that runs under a dictatorship, then it should be
Choice (A): This is the opposite of what is conveyed. because the government feels it is good. Choice (A)
Choice (B): This conveys a different idea and is, therefore,
inappropriate. 21. The given para ends with how the President will benefit
Choice (C): This can be eliminated for the same reason from the recent allegations about the illicit use of funds in
given for B. Shanghai. (D) is the only sentence which explains why he
Choice (D): Correct based on the explanation given at the would benefit from it. Choice (D)
beginning. Choice (D)
22. The author speaks of the significance of the birth of a boy
12. C is the only option that talks about what the head of the to the Japanese imperial household, the legend of the
Deutsche Borse is doing better this time around, which is throne, and the implications.
an idea that is continued from the last sentence of the given Choice (A) Correct. This option, which talks about how all
para. Choice (C) this would not make any significance to the boy himself, is
the appropriate answer.
13. Choice (A): Correct. This is the only sentence that speaks Choice B: This option could continue from the last statement
about the immediate parties who are likely to be affected, of the passage, but doesn’t make a conclusive statement.
should the new solution be put in action. Choice (C): This brings in a different idea.
Choice (B): This is ambiguous. Choice (D): This is totally unrelated to what is stated.
Choice (C): This can form a part of another paragraph. Choice (A)
Choice (D): The ‘yet’ in this option, rules it out.
Choice (A) 23. The author expresses his concern on the face of the
decline of the tiger population. Poachers and politicians
14. The given para talks about how it is very difficult to spend have shown least regard for the cause.
time in the car park beside the Giants Stadium. B is only Choice (A): Correct. This option, which talks about the
sentence that offers an ideal conclusion to the line of expected consequences, is appropriate.
thought mentioned in the para. Choice (B): This can be a part of another para.
Choice (A): This is no way related to what the para is trying Choice (C): This only reinforces the thought that there is
to convey. apathy at high levels.
Choice (B): Correct based on the explanation given at the Choice (D): This brings in a new idea. Choice (A)
beginning.
Choice (C): This is not pertinent to what is stated in the 24. The given para ends with saying how the small country
para. always bites off more than it can chew. (A) is the sentence
Choice (D): This option is rather vague. Choice (B) that gives an example of how it does this. In spite of having
very little manpower it made a generous promise to supply
15. (B) is the only sentence which talks about why Mr. Guzman 300 troops to the UN. Choice (A)
was unsuccessful earlier with his appeals against Mr. 25. The para in general talks about how motorcycles in
Pinochet which is the reason why he had to do it again. Kampala have become as common as cans in other cities.
Choice (B) And (D) is the only sentence that continues along the same
16. (D) is the only sentence that talks about the hero line of thought. Choice (D)
mentioned in the given para forming an ideal conclusion to Exercise – 17
the para. Choice (D)
Explanatory notes for questions 1 to 25:
17. Choice (A): This could be a continuation, to the paragraph,
but it cannot from an effective conclusion. 1. Statement A: This statement begins the para by stating a fact.
Choice (B): Correct. This speaks conclusively of the Statement C: This is a corroboration of what is stated in A.
indispensability of the mule packer. Statement B: “It”, here, refers to “the survey” mentioned in
Choice (C): This is not relevant to what is stated. C. Hence, ACB is the appropriate sequence.
Choice (D): This also does not logically continue the para. Statement (D) is about the centres where the studies were
Choice (B) carried out and their drawbacks while all other options are
about the spread of drug-resistant tuberculosis.
18. The passage is about the freedom of the press in Nigeria Choice (D)
then and now. The example of Punch is given to compare
the change. 2. Statement C: This statement begins the paragraph stating
Choice (A): This option, which talks about what the how the Seattle Children’s hospital in the US is a perfect
newspaper wasn’t, a decade ago, leading from the first example of how the online experience can be transformed
statement of the passage. This would help to elaborate in relatively easily.
the next paragraph how things actually were a decade ago. Statement D: ‘The hospital’ refers to the hospital mentioned
Choice (B): This option, which is about “the lawless in C, hence D is a continuation of C.
fiefdom” against journalists, is not relevant to the context. Statement B : “The site” refers to the hospital’s website
Choice (C): This option, which talks about a characteristic mentioned in B. Hence , this statement concludes the
of the newspaper, is also irrelevant. paragraph corroborating what is stated in C and D.
Choice (D): This talks about the world in general. Statement (A), which mentions another feature of a
Choice (A) different web-site., is the odd man out. Choice (A)
Triumphant Institute of Management Education Pvt. Ltd. (T.I.M.E.) HO: 95B, 2nd Floor, Siddamsetty Complex, Secunderabad – 500 003.
Tel : 040–40088400 Fax : 040–27847334 email : info@time4education.com website : www.time4education.com SM1002114/93
3. Statement C: This statement begins the paragraph stating 10. Statement A: This statement begins the paragraph by
an opinion that August is cruel month for those involved in mentioning the man idea.
education. Statement C: “These foreign countries” forms a logical link
Statement D: This statement is a logical continuation of D. with A.
Statement A: This statement follows D elaborating on the Statement D: This statement is conclusive in tone.
floundering exam boards. Hence, CDA is a sequence. Statement B, which is unrelated to the idea given in A, C
Statement B is a general statement about exams while all and D, is the odd man out. Choice (B)
others are about the disappointing performance in exams
held in August. Hence, it is out of context. Choice (B) 11. Statement B: This statement begins the para talking about
a dramatic transformation which is set to take place in
4. Statement A: This statement begins the paragraph Queensland.
mentioning how the salt laws imposed by the British, taxed Statement D: This follows B elaborating on the
the production of Indian salt. transformation.
Statement B: This statement follows A explaining how Statement A: This statement is a further elaboration of D.
Gandhiji’s defiance of the laws led to a wave of protests Hence, statements B, D, A form a logical sequence.
which led the expulsion of the British. Sentence (C) states the specialities of the landscape.
Statement D: This statement is an elaboration of ‘the Hence (C) is out of context. Choice (C)
protests’ mentioned in B.
Hence, A, B, D form a sequence. ‘The campaign’ in 12. Statement B: This statement begins the para elaborating on
statement C is ambiguous and has no logical precedent. the key to electoral success in the developed states.
Hence, C is the odd man out. Choice (C) Statement D: This statement follows D linked by the word
5. Statement A: This statement begins the paragraph talking ‘them’ which refers to the “stabilising class” mentioned in B.
about the most dramatic moment in the anti-poll tax Statement A: This statement carries forward what is stated in
campaign held in Central London. D by explaining why the stabilising class voted for Obama.
Statement D: This statement follows A mentioning the Hence, statements BDA analyse the reasons why Obama
situation which led to the campaign. was re-elected – he was able to win over people who were
Statement B: This statement concludes the paragraph committed to the stability and continuity of the political order.
mentioning about the outcome of the campaign. Hence, Sentence (C) has no relevance to the context as it talks
ADB is the sequence. While A, D and B refer to the riot in about Obama’s much publicised “hope.”
central London, statement (C), which is generic in nature, Choice (C)
is the odd man out. Choice (C)
13. Statement C: This sentence begins the para stating how
6. Statement B: This statement begins the paragraph stating young people in the Middle East and Africa have inspired
the main idea on which the paragraph is based. us in the past and are all set to do it again on Saturday.
Statement C: This is an elaboration of the campaign Statement B: This statement follows C explaining how the
mentioned in B. youth are going to inspire on Saturday.
Statement A: This statement concludes the paragraph Statement D: This statement is a logical continuation of B.
mentioning the favourable consequence of the campaign. Hence, while C, B, D talk about a movement initiated by
Hence, statements B,C, A form a sequence. the Arab youth and the purpose behind it, statement A
The one that would not affect the continuity is sentence (D). talks about a different aspect – a possibility.
Choice (D) Hence sentence (A) is irrelevant to the context.
7. Statement A: This statement, which is generic in nature, Choice (A)
begins the paragraph by giving an introduction to what
follows in the subsequent statements. 14. Statement A: This statement begins the paragraph by
Statement D: This statement follows A, presenting a mentioning the topic – the huge influx of migrant labourers
contrasting idea that it is not willpower but genes, which are into Kerala.
believed to trigger the obesity epidemic. Statement C: This statement is a corroboration of what is
Statement B: This statement is an elaboration of what is stated in A.
stated in D. A, D and B are a sequence about obesity and Statement D: This statement follows C mentioning the
the factors that trigger it. Statement C is out of context as it reasons for the huge influx.
refers to a totally different thing – one-third of American Statement (B), which is about what people from Kerala
adults are at a good weight relative to their height. who migrated to other countries of Asia and Africa did
Choice (C) during the days of the British Empire, does not form a part
of the sequence A, C and D. Hence sentence (B) has no
8. Statement C: This statement begins the paragraph by relevance to the context. Choice (B)
mentioning about antimalarial mosquitoes developed by a
British biotech company. 15. Statement A: This statement begins the para mentioning
Statement B: ‘The company’ refers to the company the topic - the garbage problem faced by Bangalore.
mentioned in C. Statement C: “The crisis” refers to the garbage problem
Statement D: This statement concludes the paragraph mentioned in A.
talking about a possible obstacle in the implementation of Statement D: This statement is conclusive in tone.
the project. CBD is a sequence. Option A is out of context Statement (B) talks about finding a place for garbage
since it states where mosquitoes lay eggs whereas all other cleaners and recyclers, which is related to the solution of the
sentences relate to raising genetically modified mosquitoes. problem. So (B) is irrelevant to the context.
Choice (A) Choice (B)
9. The passage is about Teagasc. All four statements seem to
be related; therefore they have to be closely examined to 16. Statement B: This statement begins the para by mentioning
see which of the statements could be eliminated, or, is a the main idea on which the paragraph is based – that
stand-alone. marriage is never a simple establishment in an India, which
Statement (A) and (B) are undoubtedly connected. is fractured along caste lines.
Statement (D) talks about "such a body" accused of wasting Statement A: This statement is a logical continuation of B.
by doing the "very thing" (the role as mentioned in B) ‘It’, here, refers to marriage mentioned in B.
Statement (C) talks about "field tests" (which is only one Statement C: This statement concludes stating how an inter
kind of research), impact of GM crops, and "not intended to caste marriage can be a potential trigger for violence in
...GM potatoes". This is a reference to a particular area of rural India. role of law enforcers.
research, confined to a particular variety, while all the other Statement D, which conveys a different idea, is out of
statements speak of generality. Choice (C) context. Choice (D)

Triumphant Institute of Management Education Pvt. Ltd. (T.I.M.E.) HO: 95B, 2nd Floor, Siddamsetty Complex, Secunderabad – 500 003.
Tel : 040–40088400 Fax : 040–27847334 email : info@time4education.com website : www.time4education.com SM1002114/94
17. Sentences (A), (C) and (D) (in the order DAC) talk about a 24. Statement D: This statement begins the paragraph with
comment made by Rahul Dravid and what he meant by it. Horace Greely’s statement.
Sentence (B), on the other hand, talks about the euphoria a Statement A: ‘The above statement’ refers to the statement
single medal can create, which has no relevance to the mentioned in D.
context. Choice (B) Statement C: This statement concludes the paragraph
talking about a grim possibility that tobacco consumption
18. Statement A: This statement begins the paragraph mentioning can kill 10 lakh people in the next ten years. Hence
that there has been little progress in combating malaria. statements DAC talk about the harmful effects of the
Statement D: This statement follows A talking about the consumption of tobacco products. Sentence (B) mentions
huge ampunts of money which the world spends in something totally irrelevant to the context – tobacco
combating malaria. products are the only legally available products that can kill
Statement C: This statement concludes the para stating half of their regular users if consumed as recommended by
that netsare little effective in coombating the disease. the manufacturer. Choice (B)
Sentence (B) is irrelevant to the context as it talks about
withdrawing a drug and replacing it with another, a step 25. Statement B: This statement, begins the paragraph
taken ten years ago. Choice (B) mentioning the findings of a new study that regular exercise
can heal the damage caused by a heart attack.
19. Statement B: This statement begins the para declaring that Statement A : This statement follows B as ‘the study’ refers
there has been a decline in the number of smokers in the to the study mentioned in B.
United States. Statement D: This statement follows D elaborating on
Statement D: This follows B mentioning what contributed to strenuous exercises. While statements BAD talk about the
the decline. importance of exercise in activating dormant stem cells in
Statement C: This statement concludes the para stating the heart, sentence (C) says something different – low
that the decline in adult smoking has stalled now. Hence, levels of psychological stress cause cardiovascular
statements B, D, C talk about ‘swelling the ranks’ of former diseases. Choice (C)
smokers and the reasons behind this while sentence (A)
states something entirely different – former smokers have Explanatory notes for questions 26 to 30:
to learn to enjoy life without cigarettes. So sentence (A) is
out of context. Choice (A) 26. Statement 2: This statement begins the paragraph referring to
Emperor Aurangzeb who had banned a certain musical
20. Statement A: This statement begins the paragraph reporting instrument, called pungi, as its noise was unpleasant.
a finding - that a high-fat diet may be bad for the brain. Statement 3: This statement follows 2 stating how this
Statement B: ‘Such damage’ in A refers to the damage harsh musical instrument was modified by a barber who
caused to the bring by high fat food as mentioned in A. This was also a professional musician.
statement follows A seeking to know whether exercise can Statement 1: This statement follows 3 explaining how the
act as a protection against such brain damage. modified instrument got its name.
Statement C: ‘That question; refers to the question posed Statement 4: This says how Ustad Bismillah Khan is
in B. Therefore, statements A, B and C form a sequence. synonymous with the shehnai.
Sentence (D), which begins with ‘so’ and talks about taking Hence 2314 form a logical sequence of the paragraph.
a walk, is no way related to the preceding sentence. Statement 5 suddenly talks about the life of Ustad Bismillah
Choice (D) Khan without discussing any events of his life in between.
Hence choice 5 is the odd sentence out. Ans: (5)
21. Statement A: This statement begins the paragraph
explaining how long years of isolation have transformed 27. Statement 2: The paragraph opens with discussing about
Suuu Kyi into a seasoned politician. Statement D : This good books of the hour as in choice 2.
statement is an elaboration of A. Statement 3: It proceeds to choice 3 stating what could be
Statement C: This statement concludes describing the their characteristics.
practical wisdom she has. Statement 1: This statement continues the idea listing out
Sentence (B), which says Myanmar cannot be transformed some of the works that fall under this category.
into a democratic state in the near future, is out of context. Statement 4: This statement follows 1 explaining why
Choice (B) these good books of the hour should not be allowed to
replace the true books because of their limited slots of
22. Statement B: This statement begins the paragraph usage. Hence 2314 form a logical sequence to the
explaining the power structure in dyadic relationships. paragraph. Choice 5 is totally out of place as it speaks about
Statement C: This statement is an elaboration of what is the lords and the philosophers and a total deviation from the
stated in B. subject. Hence choice 5 is the odd one out. Ans: (5)
Statement A: This statement concludes the paragraph
stating that the most serene relationships are those in 28. Statement 2: This statement opens the paragraph talking
which the power structure is accepted unquestioningly by about the dire necessities of life such as eating, sleeping
both partners in the dyad. Hence statements B, C A form a and dressing.
logical sequence. Sentence (D), which talks about Statement 1: This statement follows 2 stating that nobody
exercising authority in a society or family. Hence option (D) has a choice to shirk the above necessities whether he is a
is out of context. Choice (D) king or a labourer.
Statement 5: This statement carries forward the idea
23. Statement A: This statement begins the paragraph stating what we need to fulfil these necessities of life.
mentioning about a tea party held at Cambridge to Statement 3: This statement concludes the para pointing out
welcome students from Commonwealth countries. the darker side of life where to fulfil these necessities humans
Statement D: This statement follows A stating that the tea sometimes stoop to unethical acts. Hence 2153 forms a
party is an example of the international nature of the logical sequence of the paragraph. Statement 4 suddenly talks
University of Cambridge. about elections and voting - totally out of context to the
Statement B: This statement concludes the paragraph paragraph and is hence the odd one out. Ans: (4)
talking about the significance of tea parties at Cambridge
and explaining that the tea party stands as an example of 29. Statement 2: This statement, where the author highlights
an essentially English phenomenon. Hence A, D and B Indians' fondness for talking at length giving an example of
form a sequence. Statement (C) talks about the future of the late Krishna Menon who was the Defence Minister in
the tea party people. This aspect is irrelevant to the the cabinet of Pandit Jawaharlal Nehru, is the perfect
context. Choice (C) opening of the paragraph.

Triumphant Institute of Management Education Pvt. Ltd. (T.I.M.E.) HO: 95B, 2nd Floor, Siddamsetty Complex, Secunderabad – 500 003.
Tel : 040–40088400 Fax : 040–27847334 email : info@time4education.com website : www.time4education.com SM1002114/95
Statement 3: This logically follows 2 talking about the (A) This weakens the MD’s conclusion that all companies
length of the epics, the Ramayana and the Mahabharata. can increase profit by giving laptops.
Statement 5: This statement follows 3 describing the (B) This also shows that profits need not necessarily
epics with their arguments and counter-arguments. increase if employees have net access.
Statement 1: This statement concludes the paragraph (C) CORRECT: This does not weaken the MD’s
stating how these great epics have influenced and conclusion since it only points to the relaxed
enriched the young author looking for intellectual fodder. atmosphere at home and has no relevance to being
Hence 2351 is a logical order of the paragraph. Statement able to work at home or not.
4 talks about a specific situation in the Mahabharata, which (D) This points to a flaw in the MD’s reasoning.
is out of context. Hence choice 4 is the odd one out. Choice (C)
Ans: (4)
4. Situation: It is recommended that entrepreneurs should get
30. Statement 3: The paragraph essentially talks about the into setting up mid-segment hotels.
negative emotion 'anger'. Hence the opening of the Reasoning: Rooms in budget hotels are in great demand.
paragraph is ideal in statement 3 where it states that 'anger' The global slowdown has forced businesses to adopt
is a normal emotion and in fact a healthy emotion! austerity measures which has forced business travellers to
Statement 2: This statement follows stating the importance opt for business hotels rather than star hotels. The situation
of recognizing the signs or the triggers of anger in a cool must continue for the recommendation to be valid.
and rational way. (A) The resilience of the budget segment to the demand
Statement 4: This statement, which talks about the various does not account for the recommended growth of the
techniques and professional help you can use to contain segment.
anger, is a continuation of 2. (B) The slowdown in the overall economy has led to a
Statement 5: This statement, which discusses one such chain of events resulting in the demand for budget
technique called 'exercise' to effectively release hormones hotels going up. This is confirmation of the premises.
to contain anger, is a continuation of 4. Hence 3245 is the (C) The increase in revenue of the budget segment does
logical order of the paragraph. Statement 1 discusses provide adequate reason for the recommended growth.
about a totally different aspect called positive thinking and (D) CORRECT: With this, the demand for mid segment
realistic acceptance of other people's behaviour which is hotels looks set to continue. Choice (D)
totally different from what the other options discuss about.
Hence choice 1 is the odd one out. Ans: (1) 5. (A) Correct. When a new member replaces the old one
the total number remains the same. Hence the average age
Exercise – 18 would go up definitely if the age of the person who joins is
more and the age of the person who leaves is less.
Explanatory notes for questions 1 to 20: (B) If this is true the average age may or may not go up.
(C) If this is true the average age may or may not go up.
1. Situation: A company is experiencing decline in its revenue (D) If this is true, the average age may go up because of
and a suggestion is made to tackle the situation. the person who joined but may go down because of
Reasoning: The first set of words, ‘The falling revenue’ is the person who left. Choice (A)
both a fact and a problem. The second set of words, ‘make
a scientist …… the purchasing manager’ is a suggestion to 6. As the publishers does not actually edit the materials, he
solve the problem. cannot be held responsible, is how the argument goes. But
(A) While the first part is correct (the present condition) as the publisher, it is but natural that he is held
the second part (future condition) is incorrect. accountable. (C) is correct.
(B) The first set of words is not an imaginary condition. (A) This is wrong as the argument is not one of obeying
(C) The second set of words are not a consequence of the the law.
first set. (B) Even though an accomplice is not the person who
(D) CORRECT: While the first points to a problem the commits the crime actually, he partakes of the crime.
second is a solution proposed. Choice (D) Hence (B) is not relevant.
(C) Correct based on the explanation given above.
2. Situation: A loan application has been rejected by a bank. (D) is incorrect as the question is not one of public
Reasoning: The facts of the situation are that ice field has perception. Choice (C)
severe winters, the population is 10,000, the one
existing ice cream parlour has faced a decline in sales 7. (A) This is wrong as it does not offer an explanation as to
the previous year. The rejection of the loan application why old cassettes will not become extinct.
is probably based on these aspects mentioned in the (B) This cannot prevent the substitution of cassettes.
argument. (C) Correct – since the old favourites are available only in
(A) This could be an assumption – if the only ice cream cassettes, they cannot be wiped out by CDs.
parlour is not able to increase sales there may not be (D) This can only make cassettes redundant and hence
room for a second player. make them become extinct. Choice (C)
(B) The para mentions the size of the population. The
bank may have considered it as too small to patronize 8. The average food consumption has increased by 10 kg as
two ice cream parlours. compared to 1990. The passage attributes the increase to
(C) If the bank considers (A), (B) and (C) above as risks, greater prosperity in the country and more people being
then they would also assume that Mr. Frost didn’t see better off today. This would mean that the increased
the risks. earnings have resulted in more food purchased. The
(D) CORRECT: While the para mentions the place as inference is that people spend on food first, before any
having severe winters, and while ice cream is usually thing else. Choice (B)
eaten in warm weather, there is no indicator of how brief
(or long) the cold and warms seasons are, and this 9. The significant weakness of the suggestion arises from the
cannot, therefore be an assumption. Choice (D) fact that while boutiques are not adequate to meet
demand, big business is now in a position to do so.
3. Situation: The profit of a company increases after the Choice (D)
company gives laptops to its managers and meets the
phone and net bills. 10. The Hindu reviews only English books. It does not review
Reasoning: The MD says other companies can increase novels. So, we do not know anything about English novels.
their profits by doing the same. All the options except one All that we know is that while novels exist, the books
undermine or weaken the MD’s conclusion. reviewed in The Hindu (not being novels) do not use

Triumphant Institute of Management Education Pvt. Ltd. (T.I.M.E.) HO: 95B, 2nd Floor, Siddamsetty Complex, Secunderabad – 500 003.
Tel : 040–40088400 Fax : 040–27847334 email : info@time4education.com website : www.time4education.com SM1002114/96
imagination. Hence choice (d) is true – some English books (B) Correct based on the explanation given at the
(not being novels) do not use imagination. Choice (e) is also beginning.
true because ‘only novels use imagination’ meaning other (C) Though this may be true, the given statements are
books don’t, not necessarily that all novels do, but some do. regarding goods and services other than basic
Choice (D) necessities of life.
(D) This is a solution only for reduced supply.
11. The passage ends with an alternative possibility that a Choice (B)
good part of the reduction in pollution is due to vehicles
running on CNG. The assumption therefore is that both 18. According to the statements, antioxidants slow aging.
vehicular emission and industrial emissions are equally (A) Correct. This proves that the conclusion may not be
responsible for air pollution. Choice (D) undoubtedly true.
(B) This is outside the scope of the argument.
12. When Anand says that burglar alarms should be installed (C) ‘Healthy food’ does not mean food containing
in all houses, Vasanth interprets ‘houses’ to mean antioxidants. So (C) is in correct.
independent houses because he brings apartments into (D) Though this may be true, it does not have any bearing
the fold. Between (B) and (D), the latter is a better answer on the statements. Choice (A)
because Vasanth feels apartments are as threatened as
bungalows are. (A) is an assumption not an interpretation. 19. A survey shows that 30% acknowledged stealing and 64%
C is way off the mark. Choice (D) admitted to plagiarizing. However, since 93% were satisfied
with their ethics, it means that they did not consider it
13. Booker agrees that for the whole of 2007 upto October the unethical to steal or plagiarise. (C) is the correct answer.
ice cover in the artic was only 4 million sq. km (as against (A) There are no records to compare with the past. So (A)
13 million sq miles earlier). His claim is that now and (B) are incorrect.
(in February 2008) it is back to 13 million sq.km. It is clear (D) As what is an unacceptable level is not known (D) is
that he ignores the fact that it is winter now. So choice (C) not correct. As there are no comparative figures of
fits in what Booker has done. Choice (C) students considering stealing or plagiarizing as
unethical. Choice (C)
14. A nuclear family is a unit containing father, mother and their
children. Thus, though all the four are buried together, there
must also be evidence that all of them belonged to the 20. Discount is the amount reduced from the intended selling
same family. So (B) is the correct answer. price, that is, the marked price of a product. When a
(A) Since the single grave then indicates that the society manufacturer wants to cut losses, he reduces the selling
of the time considered them a unit. It is not necessary price from the marked price by offering a discount. If he
that the family consisted of only these four, as with or does not want to incur losses, he has to sell the articles at
without others, it is a nuclear family. So this option is the cost price. Hence he has to offer a discount which will
incorrect. be equal to the difference between the marked price and
(B) Correct based on the explanation given at the the cost price. Hence (C) is correct.
beginning. (A) This may result in a loss in the marked price such that
(C) his is not a necessary condition.
the difference between it and the cost price was less
(D) It is not necessary that others buried in the same site
must be relatives. Hence, this option is incorrect. than the cost price.
Choice (B) (B) This will result in a loss.
(C) Correct based on the explanation given at the
15. (A) Correct. If this were true, it means there must have beginning.
been 3000 younger and 3000 older women in the (D) This will result in profit but the question is only about
study. As 53% of 3000 who had a chance of getting just avoiding losses and not earning a profit.
cancer is less than 30% of 6000, the conclusion is Choice (C)
found incorrectly drawn.
(B) does not affect the conclusion as the argument is Exercise – 19
between the relative merits of physical exercise and
good sleep. Explanatory notes for questions 1 to 20:
(C) is outside the scope of the argument as it is not about
the risk associated with the quantum of sleep. 1. Statement a is clearly the author’s opinion, hence
(D) does not affect the conclusion. Choice (A) judgement-J.
Statement b is a fact since student enrolment can be
16. The conclusion that thin models lure more people towards
verified-F.
a product is a misconception can be true on the basis of
what is given as the premises only if women, about whom Statement c is again a fact as it is possible to verify if many
the premises are, make a majority of buying decisions. are able to pass the test-F.
Hence (D) is the correct answer. Statement d : Despite words like ‘reposed faith’, the
(A) This is not an assumption necessary to come to the statement is intended to convey the fact that the staff do
said conclusion. not educate their children at the Vidyapeth-F. Hence JFFF.
(B) This deals, again, with women layers and not all Choice (D)
layers.
(C) This is not an assumption as the argument itself is 2. Both parts of the sentence are observations of fact – F.
only about models. Statement b reports what someone said, hence it is
(D) Correct based on the explanation given above. a fact – F.
Choice (D) Though the beginning leads one to assume that sentence c
is an inference, it merely reports facts F.
17. The conclusion is that once there is a fall in demand, it will
lead to cut in production eventually resulting in lay-off, thus Sentence d is also a fact since it presents the writer’s
creating unemployment. This again will lead to further fall in observation-F.
demand and the vicious circle may not end. (B) shows that Both parts of sentence e also convey facts – F
cut in production will lead to fall in supply and hence Sentences a, c and e appear like inferences but they are
enhance demand which will bring the economy back on more in the nature of conveying information and no
rails so (B) is the correct answer. understanding is derived from the observations therein.
(A) If this were true, it would result only in further fall in Hence FFFFF. Choice (A)
demand.
Triumphant Institute of Management Education Pvt. Ltd. (T.I.M.E.) HO: 95B, 2nd Floor, Siddamsetty Complex, Secunderabad – 500 003.
Tel : 040–40088400 Fax : 040–27847334 email : info@time4education.com website : www.time4education.com SM1002114/97
3. Sentence a conveys information about the past and so is 9. The second half of sentence a is a fact (Amul to cut
open to verification – F. supplies by 25%) while the first half is the understanding
Sentence b draws a comparison between the past and the derived from it (toast will go without butter, You will blame
present and reflects the author’s opinion-J. the monsoon for the shortfall) – I
Sentence c is factual since the author merely says that one’s Statement b reports what is happening in the market,
perception leads to various interpretation of actions – F. hence fact – F.
What one could do is the author’s suggestion and so d In statement c, delayed monsoon is a fact based on which
becomes a judgement-J. the author infers that milk production will not rise – I
Statement e conveys the author’s opinion and as such in Statement d reports figures and so is a fact – F.
judgement - J. Hence FJFJJ Choice (D) In statement e, reports of adulteration is a fact; so, the
author infers, people would opt for trusted brands – I.
4. Despite the use of the idiom (set the cat among the Hence IFIFI. Choice (B)
pigeons), statement a merely reports what the Chief
Justice has done and so is verifiable and is a fact – F. 10. Statement a reports two facts – pockets bulging and
The words ‘This caused’ in statement b shows that the companies loosening purse strings. Neither of it is not an
understanding in B (… the Law Minister to … revisit Article understanding derived from the other though they are
311…) is based on the fact stated in a, hence it is an cause and effect; hence fact F.
inference – I. Statement b is a fact as it reports what companies are
Words like ‘whole body-politic’ and ‘deserves to be tackled doing – F.
effectively’ show that an opinion is involved, hence c The second half of statement c (companies paying
judgement – J. bonuses) is a fact from which the author understands that
Although statement d is structured as a question, it is they are ‘buoyed by signs of recovery’, hence inference – I.
rhetorical and reflects the author’s opinion that corrupt The condition in the past months is a fact, the change
practices of government officials have the backing of occurring now is also a fact; the reason for the change in
politicians – J. inferred by the author based on the changes, hence
Statement e is a widely accepted perception and hence inference – I. The current scenario being a contrast to the
fact – F. Hence FIJJF. Choice (C) past is a fact – F. Hence FFIIF. Choice (C)

5. Statement a reports a current event and hence is verifiable 11. It is the author’s opinion that it is consumers’ desire to save
- so a fact – F. that is causing fear to retailers, hence judgement J.
Statement b is the author’s opinion that the celebrations Statement b (shoppers will manage…) is the author’s
‘rival’ the Olympics – J. opinion – J.
Both the expression ‘seem lavish’ and ‘a great deal to Statement c reports the findings of the research, hence fact
celebrate’ are personal opinions of the author, hence – F.
judgement J. In statement d, the first half is a fact while the second half is
‘Is China the third most powerful economy?’ is open to the understanding derived from the facts, hence inference – I.
verification, hence fact – F. What the analysts believe in statement e is a fact, based on
Statement e again is a widely accepted truth and hence a which the author infers that the companies are on the
fact F. Hence FJJFF Choice (B) defensive – I. Hence JJFII. Choice (D)

6. Though the sentence structure (as a question) appears to 12. The commonality the author finds can be accepted by all,
be a suggestion, it is merely presenting alternatives and so hence fact – F.
is a fact – F. What the study reveals is verifiable, hence fact – F.
Sentence b defines two terms and as such is fact – F. The idea that jokes are made ‘to whittle her down to size’ is
Sentence c again reports the practices in the developing clearly the author’s opinion. It is neither a fact nor based on
country and the reason for them; both of which are facts, one – J.
no personal opinion or understanding is seen – F. The first part of statement d is a conclusion derived from
Sentence d is the author’s opinion of what should be done the fact in the second half. Hence an inference – I.
if something else happened - J. Statement e appears to be an opinion but the second half
Sentence e reports companies’ current practices – F. of the sentence is also an observation the author reports,
Hence FFFJF. Choice (A) hence fact – F. Hence FFJIF. Choice (B)

7. The focus of statement a is on reporting the fact of Obama 13. Statement a is a judgement not an inference because on
wining the Nobel Prize – F. the basis of one arrest, the author puts India in the same
Sentence b brings in the author’s opinion in juxtaposing the league as countries where women terrorists are familiar
President winning the peace prize with America deciding to (meaning seen quite often). This is his opinion and not
bomb the moon (thereby making the award seem ironical) logically inferred – J.
– J. Statement b is a fact since it merely reports a news – F.
Statement c also reflects the author’s opinion of the event – J. The suggestion that a woman may be involved in an attack
Statement d reports what the committee felt, something is a fact based on which the author concludes that counter
verifiable and hence a fact – F. terrorism efforts have to deal with a new face – I.
Statement e is a judgement because the author has What researchers say is a fact – F.
decided that ‘Obama has failed’ although the results of the In Statement e, the word ‘so’ makes it seem an inference
debate on the bill is still pending. Had the author inferred but in reality, the sentence merely reports a fact – this is
that Obama would fail based on the decision on the bill, it not the first-time women are involved in terrorism – F.
would have been an inference. As given it is a judgement – Hence JFIFF. Choice (A)
J. Hence FJJFJ. Choice (D) 14. The author infers that times are bad from the manner of
talking of the representatives of art market – I.
8. When the author says ‘…is expected to be …’ it is clearly Statement b reports a couple of facts about what is
based on some information and is hence a fact – F. happening in India and the world – F.
The data presented in sentence b makes it a fact – F Statement c reports what emerged from the panel
In statement c, the presence of the NRIs the world over is discussion, hence fact – F.
a fact, based on which the author believes there is a Statement d is the author’s opinion of what the emergent
potential market for coffee export – hence inference – I. art market did and is doing, hence judgement – J.
In statement d, the author says what India should do, The first part of statement e is an understanding (…a few
which is his opinion, hence judgement – J. heartening signs) derived from what has happened
Statement e is a fact since it reports what can be done recently (a fact) hence inference – I.
(prevalent practice) – F. Hence FFIJF. Choice (D) Hence IFFJI. Choice (D)
Triumphant Institute of Management Education Pvt. Ltd. (T.I.M.E.) HO: 95B, 2nd Floor, Siddamsetty Complex, Secunderabad – 500 003.
Tel : 040–40088400 Fax : 040–27847334 email : info@time4education.com website : www.time4education.com SM1002114/98
15. Statement a is a fact since the focus is on how the Statement b – what a number of bankers told the RBI
restaurants are doing, and both parts of the sentence governor – are reason that led to the given statement (most
report observations – F. banks are likely to maintain status quo). Hence an
In Statement b, ‘there is tangible evidence’ is a fact on the upstream argument. – U.
basis of which the author infers that purse strings are being Statement c, what the CEO of Standard Chartered Bank
loosened; hence inference – I. said, shows why most banks are not likely to bring down
Statement c is the author’s opinion (…not holding back…in their lending rates. This has led to the RBI’s moral suasion,
anticipation….) hence judgement – J. hence an upstream argument – U.
What the CEO claims is open to verification, hence fact – F.
Statement e is also a fact since ‘footfalls’ and things being Statement d gives reason from past experience why banks
are verifiable – F. will not reduce lending rates. The reason leads up to the
Hence FIJFF. Choice (C) main statement, hence upstream – U.
Statement e – what ICICI has done is consequent to and in
16. On the basis of the fact that ‘top players have come response to the RBI’s moral suasion; hence downstream – D.
together’, the author infers that competition has given way UUUUD Choice (D)
to harmony – I.
Statement b merely reports what the retailers are doing, 2. The given statement is the HRD minister’s proposal to
hence fact – F. make class X board exam optional.
The parallel the author draws in sentence c is his opinion, Statement a gives the reason behind the minister’s act. The
hence judgement – J. reason leads up to the act. Hence an upstream argument – U.
The hope of the retailers, in sentence d, is a fact since it is Statement b opines on the implication of the minister’s move,
based on some information – F. the consequence, hence a downstream argument – D.
Sentence e is also a fact since it merely reports the players Statement c is about the ‘other examination reforms’ which
clarification – F. Hence JFJFF. Choice (B) is irrelevant to the proposal to make examination for class
X optional – I.
17. The author concludes that the Hindi film industry cannot be
rid of piracy based on the fact that its tentacles are spread – I Statement d – what the Kerala education minister said – is
Statement b is a fact since one can verify whether efforts also irrelevant to the main issue – I.
are being made – F. Statement e – what the minister would like to do in the long run
Both the government and industry stepping up action is a has no bearing on the given statement, hence irrelevant – I.
fact and the author attributes this to a realization, hence UDIII Choice (B)
inference – I.
Statement d reports what Maharashtra has done recently. 3. The given statement concerns the tea board’s intention to
Hence fact – F. move to countries in the South of Africa having established
Statement e is the author’s opinion of what this law will a presence in the north of the continent.
achieve. It is subjective, so J. Hence IFIFJ. Choice (D) Statement a – the tea industry’s preparatory acts in the
past 5 years – has led to its foray into Africa. Hence
18. The author’s opinion is evident in the use of words like upstream U.
‘highly inefficient’ and ‘badly needs’ hence judgement – J.
Statement b – what the position was previously is irrelevant
From the urgent need, the author infers that the first person
the present scenario – I
to find a better bulb would make millions – I.
Based on the announcement made by Philips, the author Statement c – the knowledge of how much tea South
infers that Philips may be the one to make millions – I. Africans consume has led the tea industry to tap into this
The author infers that the prize money is ‘beside the point’ market. Hence upstream – U.
because the winner will make a lot more by being the first Statement d – the exhibition in South Africa is a
in the consumer market – I. consequence of the decision to venture into the region;
The author infers the reason why the L prize has attracted hence downstream – D.
attention – I. Hence JIIII. Choice (C)
Statement e – the absence of good tea in South Africa –
has led to the tea industry’s entry into this market; hence
19. The author concludes that doctors have changed track
upstream – U. UIUDU Choice (D)
because people are switching to alternative therapies – I.
Based on the rising popularity of Unani, Homeopathy and 4. The given statement says that banks withdraw their money
Ayurveda, the author infers that allopathy is no longer the in mutual funds to show liquidity at the end of a quarter.
in thing – I. Statement a is on the mutual fund industry’s response to
Ten students vying for one seat is verifiable, hence a fact – F. the move. The response is in consequence to the main
Some having dollar dreams while others believing in the statement, hence downstream – D.
process are open to verification, hence a fact – F. Statement b shows that this happens at every quarter. It
What the students said is fact – F. thus lends lateral support to the statement – L.
Hence IIFFF. Choice (A) Statement c gives the reason behind the move in the main
statement; hence upstream – U.
20. Statement a is a fact since ‘the historic first’ is acceptable Statement d also explains why banks act as they do hence
to all – F. upstream – U.
How it is being hailed can be verified and the project getting Statement e also explains the banks’ action, hence
terminated is also a fact – F. upstream – U. DLUUU Choice (A)
Statement c also reports something going on, hence a
fact – F. 5. The main statement says that the Asian countries have
What the popular reaction is, is a fact – F. overtaken the US and European nations as buyers of
Statement e is the author’s opinion of how we see the Indian spice.
world, hence judgement – J. Hence FFFFJ. Choice (D) Statement a refers to an increase in consumption of spices
in some of the Asian countries which has led to the change
Exercise – 20 noted in the main statement, hence upstream – U.
Statement b talks of West Asian countries that have always
1. The given statement says that a few banks many fall in line bought Indian spices. Hence, their action has had no
with the RBI’s moral suasion and cut their lending rates. impact on the change noted in the main statement – I
Statement a gives the RBI governor’s attitude to the matter Statement c gives a reason for the increased demand from
which has preceded and led to the RBI’s moral suasion. Asian countries; hence it leads to the increased demand in
Hence it is an upstream argument-U. the main statement or an upstream argument – U.

Triumphant Institute of Management Education Pvt. Ltd. (T.I.M.E.) HO: 95B, 2nd Floor, Siddamsetty Complex, Secunderabad – 500 003.
Tel : 040–40088400 Fax : 040–27847334 email : info@time4education.com website : www.time4education.com SM1002114/99
Statement d is what the spice Board Chairman said which 11. The main statement refers to the re-release of an old film in
shows what they did to increase the demand from Asian colour, which raises questions about the nature of art and
countries, hence upstream – U. revisiting it in this manner.
Statement e is irrelevant since it refers to what the US buys Statement a says art is a part of society and as society
– I. UIUUI Choice (C) changes art must change. It thus gives reasons for the act
mentioned in the main statement, hence upstream – U.
6. The main statement is about granting geographical Statement b – what will happen to the remake – is a
indicator (GI) patent to Tirupati laddus. consequence of the main statement, hence downstream – D.
Statement a explains the basis for awarding geographical Statement c refers to what Mona Lisa’s painting would be if
indications (GI) patent rights. Hence it leads to the it had a grin instead of a smile. Since this is a change to the
argument whether GI patent rights should be given to original, not a revisit or modification, it is irrelevant – I
Tirupati laddus, hence upstream – U. Statements d and e are responses to the change
Statement b refers to another product that has GI patents. mentioned in the main statement, hence downstream – D.
Hence it offers lateral support-L. UDIDD Choice (B)
Statement c explains why GI patent is required. It thus
leads to the argument in the main statement – U. 12. The main argument is against dropping the 50 over cricket
Statements d and e give two reasons why the concept of GI in favour of the Twenty 20.
exists; hence both are upstream – U. ULUUU Choice (D) Statement a gives a reason or justifies the decision to drop
7. The main statement refers to BBC modifying nursery the ODI, hence it leads up to the main argument – U.
rhymes to spare kids negative feelings. Statement b compares the ODI to test cricket and indirectly
Statement a – the BBC’s reasoning for its action – leads to lends support to the argument. Hence a lateral argument – L.
the action in the main statement – U. Statement c looks at the consequence of the challenge
Statement b refers to other well-intentioned acts in society. posed by twenty - 20 hence downstream – D.
Hence it lends lateral support – L. Statement d, like a, gives reason for scrapping ODI, hence
Statement c is a response to the BBC’s move in the main upstream – U.
statement; hence downstream – D. Statement e also justifies the action in the main statement –
Statement d gives a justification for modifying nursery rhymes U. ULDUU Choice (D)
– the reason behind the act in the main statement – U.
Statement e, what nursery rhymes do to children, is 13. The main statement refers to the corporate Affairs Minister
irrelevant since it is not connected to the modification of the asking corporations not to pay vulgar amounts of money to
rhymes – I. ULDUI Choice (B) top executives. The argument is that you should show
some sensitivity to the plight of the common man.
8. The main statement is concerning the rotation policy recently Statement a gives the mismatch between the CEO’s salary
adopted by the BCCI while allotting matches to cities. and the per capita which could be a reason behind the
Statements a, b and c give different reasons for adopting Minister’s request. Hence upstream – U.
the rotation policy. These have led to the policy being Statement b is irrelevant because the limitation sought in
adopted hence upstream arguments- U. America is for a different reason – the bank had failed and
Statement d is a response to the policy being adopted; a sought a bail out, hence it is irrelevant – I.
consequence of the action; hence downstream – D. Statement c is a response to the argument in the main
Statement e gives one more reason why the rotation policy statement, hence downstream – D.
is adopted – U. UUUDU Choice (D) Statement d is irrelevant since it refers to the manner in
which executive compensations are decided, it looks at
9. The main statement refers to the penalty levied by private whether they deserve it or not (link between performance
and MNC banks on prepayment of loans. The RBI and salary) which is not an issue raised in the main
disapproves of the practice and has called for self statement – I.
regulation by banks. Statement e raises a question about high executive
Statement a, a condemnation of the banks’ practice, has led compensation and thus leads to the main statement – U.
to the RBI’s disapproval. Hence an upstream argument – U. UIDIU Choice (C)
Statement b looks at what could be the consequence of
scrapping the prepayment penalties. It flows from the main 14. The main statement is in favour of monitoring money
argument, hence downstream – D. flowing into the country as a means to check terrorism.
Statement c refers to the practice in the US where a Statement a is the response of critics to the move in the
customer gets lower interest rates if he agrees to a non- main statement, hence downstream – D.
prepayment clause. This does not lend support to the main Statement b refers to other countries that have acted in this
argument in favor of scrapping the prepayment penalties. manner, thus offering lateral support to the move – L.
Therefore, it is irrelevant (not lateral) – I. Statement c gives reason why the move in the main
Statement d voices argument justifying the penalty for statement is needed, thus an upstream argument – U.
prepayment. It thus leads up to the argument in the main Statement d looks at what would happen if the move in the
statement – U. main statement is implemented – D.
Statement e, in effect. says banks should be conservative/ Statement e recounts what is currently happening (and
and hence justifies penalty – U. UDIUU Choice (C) hence the need for the move in the main statement) which
leads to the main statement – U. DLUDU Choice (D)
10. The main statement criticizes Shashi Tharoor for making
fun of his own party’s austerity policy. 15. The main statement refers to the controversy raised by the
Statement a gives another instance of Tharoor acting in a court’s decision to release a terrorist on compassionate
manner similar to that given in the main statement. It thus grounds.
lends lateral support to the argument – L. Statement a gives a reason for the court acting as it did.
Statement b – the call for his resignation – is a consequence Hence an upstream argument – U.
of the main statement; hence downstream – D. Statement b is irrelevant since it goes to the grounds on
Statement c refers to members of the House of commons which he was convicted in the first place – I.
and compares Indian politicians to them. But the Statement c looks at a consequence of the action in the
comparison is in the possession or lack of humour which is main statement – hence downstream – D.
irrelevant – I. Statement d is irrelevant since it is about Gaddafi, the man
Statement d discusses the austerity drive of the party which who sought the terrorist’s release. It is not relevant to
is irrelevant to the main statement which is about Shashi justice or setting him free – I.
Tharoor’s action, hence irrelevant – I. Statement e questions the wisdom of the action in the light of
Statement e is also about austerity and humour, hence what could be the consequences, hence downstream – D.
irrelevant- I. LDIII Choice (A) UIDID Choice (A)
Triumphant Institute of Management Education Pvt. Ltd. (T.I.M.E.) HO: 95B, 2nd Floor, Siddamsetty Complex, Secunderabad – 500 003.
Tel : 040–40088400 Fax : 040–27847334 email : info@time4education.com website : www.time4education.com SM1002114/100
16. The main statement justifies the move to make mandatory, Statement b refers to other factors that would support
a three month waiting period before couples can move simple living; hence lateral support – L.
divorce proceedings. Statement c gives another reason for a simple lifestyle – L.
Statement a gives a reason why the waiting period is Statement d contradicts the main statement and gives
justified. It thus leads to the action cited in the main reasons against it, it is a response to the move hence
statement – U. downstream – D.
Statement b looks at the implication of the move to couples Statement e is irrelevant since the argument is not about
waiting for divorce – D. wealth – I. ULLDI Choice (C)
Statement c cites social concerns over marital discord – it
thus lends lateral support to the move mentioned in the 19. The main statement justifies the move to recall Dravid for
main statement – L. ODI.
Statement d is irrelevant since legislators or law makers are Statement a gives one reason for arriving at the opinion in
the ones behind the move in the main statement not the main statement – U.
politicians – I Statement b is an opinion on what the selectors should
Statement e gives a reason why the state must support have done, which is a response to the event – D.
family; it thus offers lateral support – L. UDLIL Statement c is also a response to the selector’s decision – D.
Choice (D) Statement d gives reason why the author speaks in favour
of Dravid, hence it leads to the main statement – U.
17. The main argument looks at the move to check employees Statement e recounts the practices in Australia to support
socializing on the web during office hours and finds that it is the author’s view in the main statement, hence a lateral
much ado about nothing. statement – L. UDDUL Choice (D)
Statement a is an upstream argument since it gives the
reason why the author considers it a much ado about 20. The main statement is about the need to overhaul the
nothing – U. educational system and against the move to do away with
Statements b, c and d give different reasons why people the examination.
socialize at work. These thus lead to socializing in office Statement a refers to other countries but it is in relation to
hours which has provoked the reaction. Hence upstream detention not examination. Hence irrelevant – I
arguments – U. Statement b is about the responsibility of schools not the
Statement e points to what may be needed to implement educational system – I
the decision, hence downstream – D. UUUUD Statement c gives a reason for the objection raised in the
Choice (B) main argument, hence upstream – U.
Statements d and e both give reasons why the move will do
18. The main statement talks about a move for a simple more harm than good, they lead to the conclusion arrived at
uncluttered life. in the main statement, hence they are upstream – U.
Statement a refers to people in the past who have IIUUU Choice (A)
advocated a simple life. It thus gives reason for the move in
the main statement – U.

Triumphant Institute of Management Education Pvt. Ltd. (T.I.M.E.) HO: 95B, 2nd Floor, Siddamsetty Complex, Secunderabad – 500 003.
Tel : 040–40088400 Fax : 040–27847334 email : info@time4education.com website : www.time4education.com SM1002114/101

You might also like